[obm-l] Como é que faz?

2004-07-15 Por tôpico Marcelo Souza
Alguém poderia me mostrar como faz os seguintes problemas?

1. Achar todas as triplas (x,y,z) de inteiros positivos satisazendo o sistema de equações

x^2=2(y+z)
x^6=y^6+z^6+31(x^2+y^2)

2. Resolver a seguinte equação diofantina:

x+5y+10z+25w+50k=100

Alguém poderia me dar algumas referências de livros que ensinem técnicas para resolução de equações diofantinas quando o número de incógnitas é grande?

[]'s, Marcelo.MSN Messenger: converse com os seus amigos online. Instale grátis. Clique aqui. 
=
Instruções para entrar na lista, sair da lista e usar a lista em
http://www.mat.puc-rio.br/~nicolau/olimp/obm-l.html
=


[obm-l] Isoperimétrica

2004-05-20 Por tôpico Marcelo Souza
Alguém conhece uma forma elementar (sem uso de cálculo, se existir) para 
demonstrar a desigualdade isoperimétrica?

- Seja C uma curva fechada, simples e de comprimento L. Seja A a área da 
região limitada por C. Então
 L^2-4piA=0,
donde ocorre a igualdade se e só se C for um círculo

obrigado
[]'s, Marcelo
_
MSN Messenger: instale grátis e converse com seus amigos. 
http://messenger.msn.com.br

=
Instruções para entrar na lista, sair da lista e usar a lista em
http://www.mat.puc-rio.br/~nicolau/olimp/obm-l.html
=


RE: [obm-l] Fatoração ( IMO )

2004-05-09 Por tôpico Marcelo Souza


From: "Fabio Contreiras" [EMAIL PROTECTED] 
Reply-To: [EMAIL PROTECTED] 
To: [EMAIL PROTECTED] 
Subject: [obm-l] Fatoração ( IMO ) 
Date: Sun, 9 May 2004 14:32:34 -0300 
 
Alguem tem ideia de como fatorar isso? Um Abraço! 
 
 

( x + y )^7 - ( x^7 + y^7 ) 
x^7+y^7=(x+y)(x^6-x^5y+x^4y^2-...-xy^5+y^6)
Daí você põe (x+y) em evidência pra obter:
(x+y)^7-x^7-y^7=(x+y)[(x+y)^6-x^6+x^5y-x^4y^2+...+xy^5-y^6)]
A rigor,já está fatorado, mas, você pode "ajeitar" a expressão dentrodo [].
[]'s, Marcelo.
MSN Messenger: converse com os seus amigos online. Instale grátis. Clique aqui. 
=
Instruções para entrar na lista, sair da lista e usar a lista em
http://www.mat.puc-rio.br/~nicolau/olimp/obm-l.html
=


[obm-l] Problema legal

2004-04-27 Por tôpico Marcelo Souza
Olá pessoas 

Alguém poderia me dar uma mãozinha neste probleminha

1. João resolve equações quadráticas. Resolvendo a equação x^2+p_1x+q_1=0, ele encontra duas raízes reais p_2, q_2, p_2q_2. Então ele resolve x^2+p_2x+q_2=0 e assim por diante...
Até quando este exercício se repetirá, sabendo que João não conhece números complexos?

obrigado
[]'s, Marcelo.MSN Messenger: converse com os seus amigos online. Instale grátis. Clique aqui. 
=
Instruções para entrar na lista, sair da lista e usar a lista em
http://www.mat.puc-rio.br/~nicolau/olimp/obm-l.html
=


[obm-l] Um problema de números

2004-04-18 Por tôpico Marcelo Souza
Olá, alguém poderia me ajudar com este problema:

1. Seja 133^5+110^5+84^5+27^5=n^5. Qual o valor de n?

obrigado 
[]'s, Marcelo.MSN Messenger: converse com os seus amigos online. Instale grátis. Clique aqui. 
=
Instruções para entrar na lista, sair da lista e usar a lista em
http://www.mat.puc-rio.br/~nicolau/olimp/obm-l.html
=


[obm-l] Três problemas

2004-04-11 Por tôpico Marcelo Souza
Alguém me dá uma mão aki ...

1 Calcule 

 /\
|--- 
 | F.ds ,
 |
\/

onde S={(x,y,z)\in R^3| x^2+y^2+z^2=1} e F:R^3-R^3 é dado por F(x,y,z)=(x^4.e^(x^2+y^2),cos(xyz),1+y^3.z^5)

2. Determine as coordenadas do centróide de S, one S={(x,y,z)\in R^3|z^2=a^2(x^2+y^2),0=z=b}, a0 e b0}

3 - Seja B um sólido em R^3. Seja S a superfície formada pelos pontos da forma u/(sqrt(I(B,u))), onde u é o vetor unitário e I(B.u) é o momento de inércia de B em relação ao eixo definido por u. Mostre que S é um elipsóide.

obrigado []'s, MarceloMSN Messenger: converse com os seus amigos online. Instale grátis. Clique aqui. 
=
Instruções para entrar na lista, sair da lista e usar a lista em
http://www.mat.puc-rio.br/~nicolau/olimp/obm-l.html
=


[obm-l] f(x) e f'(x)

2004-02-04 Por tôpico Marcelo Souza
Suponha pum polinomio de quinto grau em x. Como demonstro que se toda raiz de p(x) é real, entaump'(x) tem 4 raizes reias (e p''(x)tem 3 raizes reais...)

[]'s, M.MSN Messenger: converse com os seus amigos online. Instale grátis. Clique aqui. 
=
Instruções para entrar na lista, sair da lista e usar a lista em
http://www.mat.puc-rio.br/~nicolau/olimp/obm-l.html
=


RE: [obm-l] CN

2004-02-01 Por tôpico Marcelo Souza




Um grupo de alunos faz prova numa sala. Se saírem do 
recinto 10 rapazes, o número de rapazes e moças será 
igual. Se, em seguida, saírem 10 moças o número de 
rapazes se tornará o dobro do número de moças. Sendo r 
o número de rapazes e m o número de moças podemos 
afirmar que 2r+m é igual a: 
(A) 60 (D) 90 
(B) 70 (E) 110 
(C) 80 


Se sairem do ambiente 10 rapazes temos
r-10 = m, pois o numero de rapazes será igual ao de moças.
Se logo em seguida, sairem 10 mocas, ficaremos com m-10 moças, cujo dobro sabemos que é o numero de rapazes que restaram (r-10), logo
2(m-10) = r-10 = m
tiramos dai que m=20, e com isso, r=30, logo 2r+m = 2*60+20 =80.
[]'s, Marcelo

MSN Messenger: converse com os seus amigos online. Instale grátis. Clique aqui. 
=
Instruções para entrar na lista, sair da lista e usar a lista em
http://www.mat.puc-rio.br/~nicolau/olimp/obm-l.html
=


[obm-l] problema

2003-10-11 Por tôpico Marcelo Souza
Alguém poderia me ajudar

O numero natural n tem seus divisores x1,x2,x3...,xk ordenados de forma que 
x1x2x3...xk. Ache todos os n tq x5^2+x6^2-1=n

[]'s

_
MSN Messenger: instale grátis e converse com seus amigos. 
http://messenger.msn.com.br

=
Instruções para entrar na lista, sair da lista e usar a lista em
http://www.mat.puc-rio.br/~nicolau/olimp/obm-l.html
=


Re: [obm-l] Teoria dos numeros

2003-09-15 Por tôpico Marcelo Souza
a) a(a^2-1)
Se a e impar entao
a^2==1 mod 8
e como (a-1)a(a+1) são tres inteiros consecutivos, temos que 3 tb o divide, 
logo 24 divide o produto

b)
Mesmo esquema
a^2==1 mod 8
b^2==1 mod 8
a^2-b^2==0 mod 8

obs.: Considere a=8k+r onde 0=r8

e eleve ao quadrado para os casos impares (para nao fazer mta conta use ao 
inves de r=5, r=-3 (por ai), que fica bem resumido, dai vc observar que o 
quadrado de um numero inteiro impar e congruente a 1 mod 8).

[]'s, Marcelo.

From: Henrique Patrício Sant'Anna Branco [EMAIL PROTECTED]
Reply-To: [EMAIL PROTECTED]
To: OBM [EMAIL PROTECTED]
Subject: [obm-l] Teoria dos numeros
Date: Sun, 14 Sep 2003 20:37:26 -0300
Prove as seguintes afirmações:
a) Se a é um inteiro ímpar, então 24 divide a*(a^2 - 1)
b) Se a e b são inteiros impares, entao 8 divide a^2 - b^2
No caso do item b) pensei em considerar a = 4k-1 e b = 4k+1. Eu perco a
generalidade se fizer algo assim?
Grato,
Henrique.
=
Instruções para entrar na lista, sair da lista e usar a lista em
http://www.mat.puc-rio.br/~nicolau/olimp/obm-l.html
=
_
MSN Messenger: instale grátis e converse com seus amigos. 
http://messenger.msn.com.br

=
Instruções para entrar na lista, sair da lista e usar a lista em
http://www.mat.puc-rio.br/~nicolau/olimp/obm-l.html
=


[obm-l] curvas

2003-07-15 Por tôpico Marcelo Souza
Gente,
alguém pode me ajudar a resolver os problemas
1. Sejam e uma elipse e h uma hiperbole tendo focos em comum. Mostre que e e 
h se cortam perpedicularmente.
(alguém podria exibir uma solução usando derivadas, é pq estou tentando e 
não consegui)

2.Seja c:I-R^2 uma curva com segunda derivada e tq c'(t) dif de  (0,0) para 
todo t. Suponha ainda que a aceleração normal não se anule. A evoluta de c é 
a curva
a:I-R^2
 t -- (x(t),y(t))+(1/k(t)(x'(t)^2+y'(t)^2)^(1/2))*(-y'(t),x'(t)),
onde c(t)=(x(t),y(t)) e k(t) = (-y'x''+x'y'')/(x'^2+y'^2)^(3/2) (omiti t)

I) Mostre que a reta tangente a a em a(t) coincide com a normal a c em c(t).

valeu
[]'s, Marcelo
_
MSN Messenger: instale grátis e converse com seus amigos. 
http://messenger.msn.com.br

=
Instruções para entrar na lista, sair da lista e usar a lista em
http://www.mat.puc-rio.br/~nicolau/olimp/obm-l.html
=


[obm-l] integral

2003-06-19 Por tôpico Marcelo Souza
fala pessoal da lista!
Como faz pra resolver a integral
 /\
 |
 |   cos(nx)
 |  dx
 |  (x^4+1)
\/
valeu
[]'s,Marcelo.
_
Help STOP SPAM with the new MSN 8 and get 2 months FREE*  
http://join.msn.com/?page=features/junkmail

=
Instruções para entrar na lista, sair da lista e usar a lista em
http://www.mat.puc-rio.br/~nicolau/olimp/obm-l.html
=


[no subject]

2003-06-03 Por tôpico Marcelo Souza
alguem pode me ajudar com o problema

--Considere  o problema de aproximar, numericamente, a trajetória dada por 
C:R -- R3, sabendo que C(t0)=P0, C'(t0)=v0 (dados) e que

C'' (t)= a(t)c(t)

onde a:R -- R também é dada. fixe delta t maior do que zero, tn=t0+n*delta 
t

metodo 1 : P(n+1) = Pn + delta t* Vn; V(n+1) = Vn+deltat*a(tn)*Pn
metodo 2: P(n+1) = Pn + deltat*Vn; V(n+1)=Vn+deltat*a(tn+1)*P(n+1)
(i) mostre que, em ambos os metodos, Pn pertence a alfa, onde alfa é o plano 
passando por (0,0,0) e gerado por P0 e V0 (se P0 e V0 são linearmente 
independentes). Que acontece se P0 e V0 não são linearmente independentes?

_
MSN 8 helps eliminate e-mail viruses. Get 2 months FREE*. 
http://join.msn.com/?page=features/virus

=
Instruções para entrar na lista, sair da lista e usar a lista em
http://www.mat.puc-rio.br/~nicolau/olimp/obm-l.html
=


[obm-l] equação

2003-04-01 Por tôpico Marcelo Souza
Alguém poderia achar as raizes da equação (usando um computador que a mao esta dificil)

[4/(sqrt(x^2-900) + 6/(sqrt(x^(2)-400)] = 15
obrigado pela ajuda
[]'s, M.Add photos to your messages with  MSN 8.  Get 2 months FREE*.
=
Instruções para entrar na lista, sair da lista e usar a lista em
http://www.mat.puc-rio.br/~nicolau/olimp/obm-l.html
O administrador desta lista é <[EMAIL PROTECTED]>
=


[obm-l] livro

2002-12-03 Por tôpico Marcelo Souza
alguém pode me indicar um livro (que naum seja do Arthur engel - Porlbem 
solving strategies) que tenha um bom capitulo (ou livro inteiro) sobre 
equaçoes fundionais
sou grato
[]'s M.

_
MSN 8 with e-mail virus protection service: 2 months FREE* 
http://join.msn.com/?page=features/virus

=
Instruções para entrar na lista, sair da lista e usar a lista em
http://www.mat.puc-rio.br/~nicolau/olimp/obm-l.html
O administrador desta lista é [EMAIL PROTECTED]
=


[obm-l] quadrados perfeitos

2002-11-29 Por tôpico Marcelo Souza
1. Para quais valores de (a,b), temos que (a^2+4b) e (b^2+4a) são ao mesmo tempo quadrados perfeitos?
valeu
[]'s, MarceloMSN 8 helps ELIMINATE E-MAIL VIRUSES. Get 2 months FREE*.
=
Instruções para entrar na lista, sair da lista e usar a lista em
http://www.mat.puc-rio.br/~nicolau/olimp/obm-l.html
O administrador desta lista é <[EMAIL PROTECTED]>
=


Re: [obm-l] Teoria dos Números

2002-11-18 Por tôpico Marcelo Souza

Esta primeira parece ser fácil...
1. Um número A formado por trezentos dígitos iguais a um não pode ser um quadraado perfeito pqnaum eh potencia par de 3,já quetera apenas um fator 3, sendo assim naum eh quadrado perfeito.Espero naum ter errado o raciociniopareceu muito simples.
[]'s, Marcelo
From: "Roberto Gomes" <[EMAIL PROTECTED]>
Reply-To: [EMAIL PROTECTED] 
To: [EMAIL PROTECTED] 
Subject: [obm-l] Teoria dos Números 
Date: Fri, 15 Nov 2002 19:22:49 + 
 
Problemas do livro de teoria dos números do José Plínio de Oliveira Santos, que não consegui resolver. 
 
1. Pode o número A=111 formdo por trezentos 1's ser um qadrado? 
 
2. Mostrar que todo inteiro maior do que 11 é soma de dois inteiros compostos. 
 
3. Seja Un = 111...1 um número formados por n 1's. Provar que Un primo implica n primo. 
 
4. Provar que se d = mdc(a,b), então d é o número de inteiros na seqüência a, 2a, 3a, , ba que são divisíveis pr b. 
 
5. Seja p primo e M um conjunto de p inteiros consecutivos. É possível encontrar M1 e M2 subconjuntos de M tais que M1 È M2 = M, M1Ç M2 =Æ , Mi ¹ Æ de forma que 
 
P i = P j ? 
i ÎM1 j Î M2 
 
6. Seja f(x) um polinômio com coeficientes inteiros. Mostrar que se f(-1), f(0) e f(1) não são divissíveis por 3, então f(n) ¹ 0 para todo n. 
 
7. Encontrar um sistema completo de resíduos módulo 7 onde todos os elementos são números primos. 
 
8. Dado um primo p é sempre possível encontrar um sistema completo de resíduo módulo p formado só por primos? Justivicar. 
 
 
Obrigado pela atenção de todos. 
 
Roberto Gomes, Recife-PE 
The new MSN 8: smart spam protection and 2 months FREE* 
=
Instruções para entrar na lista, sair da lista e usar a lista em
http://www.mat.puc-rio.br/~nicolau/olimp/obm-l.html
O administrador desta lista é <[EMAIL PROTECTED]>
=


[obm-l] Recorrência

2002-11-09 Por tôpico Marcelo Souza
Oi pessoal, como resolvo a recorrência

x_{n}=(n+1)x_{n-1}-nx_{n-2}?

me enrolei pq os coeficientes não são contantes...
falow
[]'s
Marcelo

_
STOP MORE SPAM with the new MSN 8 and get 2 months FREE* 
http://join.msn.com/?page=features/junkmail

=
Instruções para entrar na lista, sair da lista e usar a lista em
http://www.mat.puc-rio.br/~nicolau/olimp/obm-l.html
O administrador desta lista é [EMAIL PROTECTED]
=


[obm-l] quadrilátero e LG

2002-10-19 Por tôpico Marcelo Souza
Olá pessoal, 
Dado um quadrilátero ABCD e um ponto O interior a ele. Liga-se O aos vértices do quadrilatero, formando assim, quatro triangulos. Pede-se para determinar o LG dos pontos O para os quaisa soma das áreas de dois triângulos opostos seja igual a metade da área do quadrilátero.
obs: o quadrilátero não é um paralelogramo.
Dá pra ver que os pontos médios das diagonais pertencem ao LG, e depois disso, levados a suposição de acreditar que eh uma reta, mas não estou conseguindo argumntos para provar. Alguém poderia me ajudar?
Obrigado
[]'s, MarceloInternet access plans that fit your lifestyle -- join MSN. Click Here 
=
Instruções para entrar na lista, sair da lista e usar a lista em
http://www.mat.puc-rio.br/~nicolau/olimp/obm-l.html
O administrador desta lista é <[EMAIL PROTECTED]>
=


Re: [obm-l] Problema do Márcio - jogo de tv

2002-10-15 Por tôpico Marcelo Souza


Bom, acho que do jeito que eu fiz eh mais ou menos a mesma coisa, veja:
x_1+2x_2+4x_3+...+512x_10=610
onde x_i pertence a {0,1}
Ele deve ter acertado a pergunta 10, caso contrário, 'nào daria para obter 
tal pontuacao...observando tal fato, eh fácil concluir que ele acertou as 
perguntas 9,8,5,2...zerando as outras.
Falow
[]'s Marcelo

From: Gabriel Pérgola [EMAIL PROTECTED]
Reply-To: [EMAIL PROTECTED]
To: Obm-l [EMAIL PROTECTED]
Subject: [obm-l] Problema do Márcio - jogo de tv
Date: Sun, 13 Oct 2002 20:06:29 -0300

E aí pessoal,

Estava olhando o problema que o Márcio mandou para a lista:

Em um jogo de televisão, um candidato deve responder a 10 perguntas. A
primeira vale 1 ponto, a segunda vale 2 pontos, e assim, sucessivamente,
dobrando sempre. O candidato responde a todas as perguntas e ganha os 
pontos
correspondentes às respostas que acertou, mesmo que erre algumas. Se o
candidato obteve 610 pontos, quantas perguntas acertou?


E vi a solução usando número binários (colocando na base dois)..

Gostaria de saber se existe alguma outra forma de resolver este problema, e
se sim, como?

Abraço,

Gabriel

=
Instruções para entrar na lista, sair da lista e usar a lista em
http://www.mat.puc-rio.br/~nicolau/olimp/obm-l.html
O administrador desta lista é [EMAIL PROTECTED]
=


_
Chat with friends online, try MSN Messenger: http://messenger.msn.com

=
Instruções para entrar na lista, sair da lista e usar a lista em
http://www.mat.puc-rio.br/~nicolau/olimp/obm-l.html
O administrador desta lista é [EMAIL PROTECTED]
=



Re: [obm-l] Gabarito da olimpíada carioca.

2002-09-06 Por tôpico Marcelo Souza

Bom, eu fiz as questoes e:
1) A primeira deu 8/15
2) Era só fazer o gráfico das funções log(x) e 2log(5)senx que cortava em 7 
pontos.
3)a) a_2001 é par (pela congruencia modulo 2)
b) a_2002 era multiplo de 3 (congr. mod 3)
c) Sim
d) Não
4)Saía por quadrilateros inscritíveis, notando que A_2B_3 era paralelo a AB 
(ou A_2B_2naum me liguei muito em qual dos lados ele mandou colocar B_2 
ou B_3...tanto faz)
falow
Abracos
Marcelo
obs.:Tenho muita certeza dos resultados, mas posso estar errado. Caso 
esteja, peço que o pessoal da lista corrija.


From: haroldo [EMAIL PROTECTED]
Reply-To: [EMAIL PROTECTED]
To: [EMAIL PROTECTED]
Subject: [obm-l] Gabarito da olimpíada carioca.
Date: Tue, 3 Sep 2002 15:27:51 -0300

Gostaria de ver o gabarito oficial da olimpíada carioca, realizada na
PUC, dia 31/08.
Pois tenho dois alunos que participaram da competição.
Grato.
Haroldo .


_
Chat with friends online, try MSN Messenger: http://messenger.msn.com

=
Instruções para entrar na lista, sair da lista e usar a lista em
http://www.mat.puc-rio.br/~nicolau/olimp/obm-l.html
O administrador desta lista é [EMAIL PROTECTED]
=



Re: [obm-l] Olimpíadas on line

2002-08-26 Por tôpico Marcelo Souza

Parece ser legal. Talvez fosse melhor colocar uma prova com, digamos 10 
questoes e tivessemos os tais 10 dias para resolve-la e envia-las..seria 
bacana. Espero que a ideia dê certo.


From: Laurito Alves [EMAIL PROTECTED]
Reply-To: [EMAIL PROTECTED]
To: [EMAIL PROTECTED]
Subject: [obm-l] Olimpíadas on line
Date: Tue, 27 Aug 2002 01:15:12 +


Colegas da lista,

Sou coordenador do curso de matemática de uma faculdade em Belo Horizonte. 
Se desejarem, posso verificar a possibilidade de hospedar a Olimpíada 
Virtual em nosso site.Tenho o apoio de vocês ?

Laurito Alves



_
Converse com seus amigos online, faça o download grátis do MSN Messenger: 
http://messenger.msn.com.br

=
Instruções para entrar na lista, sair da lista e usar a lista em
http://www.mat.puc-rio.br/~nicolau/olimp/obm-l.html
O administrador desta lista é [EMAIL PROTECTED]
=


_
Join the world’s largest e-mail service with MSN Hotmail. 
http://www.hotmail.com

=
Instruções para entrar na lista, sair da lista e usar a lista em
http://www.mat.puc-rio.br/~nicolau/olimp/obm-l.html
O administrador desta lista é [EMAIL PROTECTED]
=



Re: [obm-l] olimpiada virtual

2002-08-26 Por tôpico Marcelo Souza

Gostei muito dessa sugestao. Achei a mais organizada, porem, naum eh uma 
coisa soh entre nós. Teríamos que ver qual professor teria paciencia de 
corrigir, estabelecer criterios de correcao. Naum eh bem simples assim.
Talvez fosse bom fazer grupos (se houvessem grupos) de estado em 
estadotipo, um pro Rio, outro pra sao paulo, etc...


Eu ando meio enrolado mas  eu topo. Nao sei se seria
uma boa , mas acho que poderia ser assim: tinhamos 10
dias para entregar, ninguem entregaria antes, depois
dos 10 dias , todo mundo envia as soluçoes para os
professores e eles apresentavam os pontos.
Talvez poderiamos formar umas duplas ou grupos  para
competicao.
Temos que discutir os niveis dos problemas.




_
Send and receive Hotmail on your mobile device: http://mobile.msn.com

=
Instruções para entrar na lista, sair da lista e usar a lista em
http://www.mat.puc-rio.br/~nicolau/olimp/obm-l.html
O administrador desta lista é [EMAIL PROTECTED]
=



Re: [obm-l] Relativamente Primos???

2002-08-25 Por tôpico Marcelo Souza



Nunca tinha ouvido falar, mas em todo caso peço ajuda.

1) Provar que 4k+3 e 5k+4 são relativamente primos, para todo inteiro k.

Isto se torna bem simples se vc usar o fato abaixo. Vou esccrever mdc(a,b) 
simplesmente como (a,b).
--Se a, b e c são inteiros (a,b)=(a,b+ac).
logo esccrveemos 
(4k+3,5k+4)=(4k+3,5k+4-4k-3)=(4k+3,k+1)=(4k+3-3k-3,k+1)=(k,k+1)=(k,1)=1.
[]'s. Marcelo


_
Send and receive Hotmail on your mobile device: http://mobile.msn.com

=
Instruções para entrar na lista, sair da lista e usar a lista em
http://www.mat.puc-rio.br/~nicolau/olimp/obm-l.html
O administrador desta lista é [EMAIL PROTECTED]
=



Re: [obm-l] Política NAO é assunto da lista. - SPAM - não vote nesse SPAMEIRO

2002-08-25 Por tôpico Marcelo Souza

Se me permitem dizer, este foi um dos piores off-topic que a lista ja 
teve.Utilizar a lista para divulgacao politica e incorreto, afinal, aqui e o 
unico lugar onde nos vemos livres de toda lavagem cerebral que os politicos 
fazem por meio da televisao e do radio. Peço apenas um pouco de respeito, so 
isso, respeito com os colegas da lista e um pouco de conveniencia. O 
objetivo da lista e bem claro-estamos aqui para discutir problemas de 
matematica! Agora e triste abrir uma mensagem e descobrir que o unico lugar 
a salvo de todas as corrupcoes foi violado bruscamente por uma mensagem 
desse tipo. Espero que as pessoas respeitem mais a nossa lista, caso 
contrario, daqui ha pouco, ela sera ate mesmo veiculo de vendas...
abraços
Marcelo


From: Eduardo Azevedo [EMAIL PROTECTED]
Reply-To: [EMAIL PROTECTED]
To: [EMAIL PROTECTED]
Subject: [obm-l] Política NAO é assunto da lista.  - SPAM - não vote nesse 
SPAMEIRO
Date: Sun, 25 Aug 2002 02:32:53 -0300

Se não tem vergonha devia ter.
Uma mula como você pode achar que política e matemática são a mesma coisa, 
mas não são.

O Manoel pode trabalhar muito no interior do Estado(nem sei qual), mas pra 
minha vida ele só trouxe SPAM. Esse é um candidato em quem eu não voto, e 
se vir alguém votando, logo falo pra não votar.

OBS:
Ser ousado é outra coisa.
Você é só sem vergonha mesmo.

[EMAIL PROTECTED] = SPAM
SPAMMER = mail bomb

  Original Message -
   From: ReNNeR
   To: [OBM]
   Sent: Sunday, August 25, 2002 12:59 AM
   Subject: [obm-l] Política também é assunto da lista.


   Meus amigos da lista,

   Acreditando que a matemática, assim como diversas outras áreas do 
conhecimento, é diretamente relacionada à estrutura pública que o pais 
possui, seja nos cargos executivos, legislativos ou judiciários, indico o 
nome do Manoel Veras como deputado estadual.
   Conheço o seu trabalho e a sua vontade para melhorar a vida das 
pessoas, principalmente no interior do Estado que ainda sofre diversos 
problemas sociais e enfrenta uma eduação de qualidade não muito boa. Apesar 
de esse não ser o dever direto de um Deputado, o Manoel sempre se preocupou 
e procurou soluções para esses tipos de problemas.
   Não tenho vergonha de ser ousado e pedir, além do seu voto, o seu 
apoio a essa candidatura, pois se trata de um Deputado que faz por merecer!



Obrigado e não esqueça: Manoel Veras - 
45145


_
Chat with friends online, try MSN Messenger: http://messenger.msn.com

=
Instruções para entrar na lista, sair da lista e usar a lista em
http://www.mat.puc-rio.br/~nicolau/olimp/obm-l.html
O administrador desta lista é [EMAIL PROTECTED]
=



Re: [obm-l] geometria analitica

2002-08-03 Por tôpico Marcelo Souza


Eu acho o livro do KLETENIK (acho que eh assim que escreve)-editora MIR mto 
bom...Se vc naum tiver problemas com ingles ou espanhol, encontra molinho 
ele no sebo...acho que portugues tem, mas eh meio dificil d achar
abracos
marcelo

From: adr.scr.m [EMAIL PROTECTED]
Reply-To: [EMAIL PROTECTED]
To: [EMAIL PROTECTED]
Subject: [obm-l] geometria analitica
Date: Sat,  3 Aug 2002 20:54:11 -0300

gostaria de saber se alguem conhece algum
livro de geometria analitica,e pudesse me
dizer, que tenha desde distancia a  pontos
ate circunf,elipse,parabola e principalmente
lugar geometrico.
ja ouvi falar do Reis e Silva,mas nao
encontro e nao sei se tem lugar geometrico.
Obrigado.
Adriano.


__
AcessoBOL, só R$ 9,90! O menor preço do mercado!
Assine já! http://www.bol.com.br/acessobol


=
Instruções para entrar na lista, sair da lista e usar a lista em
http://www.mat.puc-rio.br/~nicolau/olimp/obm-l.html
O administrador desta lista é [EMAIL PROTECTED]
=


_
MSN Photos is the easiest way to share and print your photos: 
http://photos.msn.com/support/worldwide.aspx

=
Instruções para entrar na lista, sair da lista e usar a lista em
http://www.mat.puc-rio.br/~nicolau/olimp/obm-l.html
O administrador desta lista é [EMAIL PROTECTED]
=



Re: [obm-l] Re: [obm-l] Primeira questão obm ano passado

2002-07-29 Por tôpico Marcelo Souza


A ideia do Lucas não me parece bem simples assim. O que ele fez foi Usar o 
semi-perímetro (no caso S) e a área (P) de um triângulo de lados 
(a+b),(b+c),(a+c)...a solução é bem bonita, fica imediata até se vc desenhar 
o triângulo, não é trivial esta idéia, mas eh uma boa tecnica para 
desigualdades supor que sao lados de um triangulo.
[]'s
Marcelo

From: Fernanda Medeiros [EMAIL PROTECTED]
Reply-To: [EMAIL PROTECTED]
To: [EMAIL PROTECTED]
Subject: [obm-l] Re: [obm-l] Primeira questão obm ano passado
Date: Sat, 27 Jul 2002 21:35:43 +

  Oi Duda!
Bem, eu fiz assim:
desenvolvendo fica a(a+b+c) +bc =2sqrt[abc(a+b+c)] pela desigualdade das 
medias! :)
  té
[]´s
Fê!

Lembram daquela desigualdade, sendo a,b,c0 prove
(a + b)(a + c) = 2raiz(abc(a+b+c)).

Olhem essa solução que o Lucas Mocelim me apresentou.
Chame S=a+b+c e P=abc
(a + b)(a + c) =
(S - c)(S - b) =
S^2 - (b + c)S + bc =
S^2 - (S - a)S + P/a =
Sa + P/a = 2raiz(SaP/a) = 2raiz(SP)
Só isso, não é muito mais fácil que a solução da Eureka!?
Pena que na hora ele não percebeu...

Um abraço!
Duda.

PS David Turchick, valeu pela correção da questão da imo, agora eu já
compreendi.

=
Instruções para entrar na lista, sair da lista e usar a lista em
http://www.mat.puc-rio.br/~nicolau/olimp/obm-l.html
O administrador desta lista é [EMAIL PROTECTED]
=




_
MSN Photos é a maneira mais fácil e prática de editar e compartilhar sua 
fotos: http://photos.msn.com.br

=
Instruções para entrar na lista, sair da lista e usar a lista em
http://www.mat.puc-rio.br/~nicolau/olimp/obm-l.html
O administrador desta lista é [EMAIL PROTECTED]
=


_
Chat with friends online, try MSN Messenger: http://messenger.msn.com

=
Instruções para entrar na lista, sair da lista e usar a lista em
http://www.mat.puc-rio.br/~nicolau/olimp/obm-l.html
O administrador desta lista é [EMAIL PROTECTED]
=



Re: [obm-l] desigualdade...

2002-07-21 Por tôpico Marcelo Souza

De fato essa eh a ideia que funciona, pq ficariamos com a desigualdade:
(x+y-z)(x+z-y)(y+z-x)=xyz que naum parece dificil de provarjah que
x^2=x^2-(y-z)^2
y^2=y^2-(z-x)^2
z^2=z^2-(z-y)^2
multiplicando gera a desigualdade obtida...
abracos
Marcelo

From: Johann Dirichlet [EMAIL PROTECTED]
Reply-To: [EMAIL PROTECTED]
To: [EMAIL PROTECTED]
Subject: Re: [obm-l] desigualdade...
Date: Fri, 19 Jul 2002 17:28:47 -0300 (ART)

Nao sei onde ta o erro,mas a minha soluçao
consistia em substituir os caras por
outros(a/b,b/c,c/a)e ver algo mais simetrico.


--- Marcelo Souza [EMAIL PROTECTED]
escreveu:  Ola
  Ha uma desigualdade que eh assim
  (a-1+1/b)(b-1+1/c)(c-1+1/a)=1, sendo a,b,c0 e
  abc=1
  Quando fui resolve-la deparei-me com algo
  estranho. Alguem poderia me
  apontar se errei ou naum em algum lugar?
  observe:
  [(ab-b+1)/b][(bc-c+1)/c][(ac-a+1)/a]=1 como
  abc=1
  (ab-b+1)(bc-c+1)(ac-a+1)=1
  multiplicando e substituindo abc por 1 ficamos
  com
  [b-1+ab-b^2c+bc-b+bc-c+1][ac-a+1]=
  =[ab-b^2c+2bc-c][ac-a+1]=
  =a-a^2b+ab-bc+b-b^2c+2c-2+2bc-ac^2+ac-c=1
  dai
  a+b+c-a^2b-b^2c-ac^2+ab+bc+ac=3
  A partir dai eh estranho, pq consigo mostrar
  exatamente o contrario, pela
  MAMG...naum sei se eh apenas um erro de
  inversao de sinal. Alguem poderia me
  indicar onde errei?
  obrigado
  abracos
  Marcelo
 
 
_
  MSN Photos is the easiest way to share and
  print your photos:
  http://photos.msn.com/support/worldwide.aspx
 
 
=
  Instruções para entrar na lista, sair da lista
  e usar a lista em
 
http://www.mat.puc-rio.br/~nicolau/olimp/obm-l.html
  O administrador desta lista é
  [EMAIL PROTECTED]
 
=

___
Yahoo! Encontros
O lugar certo para encontrar a sua alma gêmea.
http://br.encontros.yahoo.com/
=
Instruções para entrar na lista, sair da lista e usar a lista em
http://www.mat.puc-rio.br/~nicolau/olimp/obm-l.html
O administrador desta lista é [EMAIL PROTECTED]
=


_
Send and receive Hotmail on your mobile device: http://mobile.msn.com

=
Instruções para entrar na lista, sair da lista e usar a lista em
http://www.mat.puc-rio.br/~nicolau/olimp/obm-l.html
O administrador desta lista é [EMAIL PROTECTED]
=



[obm-l] desigualdade...

2002-07-16 Por tôpico Marcelo Souza

Ola
Ha uma desigualdade que eh assim
(a-1+1/b)(b-1+1/c)(c-1+1/a)=1, sendo a,b,c0 e abc=1
Quando fui resolve-la deparei-me com algo estranho. Alguem poderia me 
apontar se errei ou naum em algum lugar?
observe:
[(ab-b+1)/b][(bc-c+1)/c][(ac-a+1)/a]=1 como abc=1
(ab-b+1)(bc-c+1)(ac-a+1)=1
multiplicando e substituindo abc por 1 ficamos com
[b-1+ab-b^2c+bc-b+bc-c+1][ac-a+1]=
=[ab-b^2c+2bc-c][ac-a+1]=
=a-a^2b+ab-bc+b-b^2c+2c-2+2bc-ac^2+ac-c=1
dai
a+b+c-a^2b-b^2c-ac^2+ab+bc+ac=3
A partir dai eh estranho, pq consigo mostrar exatamente o contrario, pela 
MAMG...naum sei se eh apenas um erro de inversao de sinal. Alguem poderia me 
indicar onde errei?
obrigado
abracos
Marcelo

_
MSN Photos is the easiest way to share and print your photos: 
http://photos.msn.com/support/worldwide.aspx

=
Instruções para entrar na lista, sair da lista e usar a lista em
http://www.mat.puc-rio.br/~nicolau/olimp/obm-l.html
O administrador desta lista é [EMAIL PROTECTED]
=



[obm-l] teo dos numeros

2002-07-15 Por tôpico Marcelo Souza
Ola pessoal da lista
como faco para provar o seguinte:
- Prove que se n numeros sao primos entre si em pares, entao todos eles sao primos entre si.
Parece-me quea volta naum vale, mas naum consigo mostrar um contra exemplo, alguem poderia me indicar como faco?
valeu
MarceloChat with friends online, try MSN Messenger: Click Here
=
Instruções para entrar na lista, sair da lista e usar a lista em
http://www.mat.puc-rio.br/~nicolau/olimp/obm-l.html
O administrador desta lista é <[EMAIL PROTECTED]>
=


[obm-l] Analise

2002-07-01 Por tôpico Marcelo Souza

Oi pessoal, tudo bom?
Como eu faco pra provar formalmente:
- Dados dois conjuntos A e B, sobre X temos:
1.X (contem) A e X(contem)B
2. Se Y(contem)A e Y(contem)B = Y(contem)X
Prove que X=(AUB)
Obrigado
abracos
Marcelo

_
Chat with friends online, try MSN Messenger: http://messenger.msn.com

=
Instruções para entrar na lista, sair da lista e usar a lista em
http://www.mat.puc-rio.br/~nicolau/olimp/obm-l.html
O administrador desta lista é [EMAIL PROTECTED]
=



Re: [obm-l] espacial

2002-06-30 Por tôpico Marcelo Souza




qual é a resolução(detalhada) desses exercícios?


-Quatro pontos coplanares determinam um único plano?
-Considerando duas retas r e s pararlelas a um plano
A, pode existir uma reta contida em A que seja
concorrente com r ou s?
-Se dois planos são secantes, então existe uma reta de
um deles reversa a uma reta do outro?
-Se dois planos distintos são paralelos, então toda
reta paralela a um deles é paralela ao outro?
-Considere duas circunferências de raios congruentes,
centros comuns e contidas em planso diferentes.Qual é
a intersecção dessas duas circunferências?
-Uma reta num plano pode ser considerada paralela ao plano?

Oi...
Naum sei se tem uma resolução detalhada (talbez vc queira formalizar)...mas 
vc formaliza utilizando os postulados da geometria.
-A primeira, naum sei se estou errado, mas a definicao de coplanares eh 
estarem em um mesmo plano, logo se sao 4 ptos coplanares estao num unico 
plano.(se foi bem isso que entendi)
-A intersecçao das duas circunferencias eh uma resta (ja que as duas 
determinam dois planos que se intersectam)
Tem algumas que sei responder, mas pode haver alguma exceçAo que pode me 
falhar na memoria, entao prefiro deixar aí.
falow
Marcelo


_
Chat with friends online, try MSN Messenger: http://messenger.msn.com

=
Instruções para entrar na lista, sair da lista e usar a lista em
http://www.mat.puc-rio.br/~nicolau/olimp/obm-l.html
O administrador desta lista é [EMAIL PROTECTED]
=



Re: [obm-l] infinito

2002-06-29 Por tôpico Marcelo Souza

A representação por uma reta (em Matemática) é apenas teórica (reta infinita...)É que na realidade (fisicamente) uma reta eh considerada como um circunferêcia de raio infinito...fica fácil se vc pensar no chão que vc pisa (visto que a terra eh "redonda")...Agora vejase vc desenhar o ciclo trigonométrico e traçar a reta das tangentes, terá uma explicaçao para este + ou - infinito...Creio eu que ele esteja tomando as noções de cálculo. Enfim, desenhando as tangentes, temos que a tangente de 90 eh naum definida, ou, se definida, tendendo ao infinitose vc percorrer o ciclo trigonometrico de outra maneira (de 180 ate 90), vera que os arcos tenderao para infinito pelo lado negativoou seja, os infinitos sao os mesmos.Espero ter sido claro, pois isto eh meio chato de se escrever...queria conseguir ser mais formal.
falow
abracos
Marcelo

From: "adr.scr.m" <[EMAIL PROTECTED]>
Reply-To: [EMAIL PROTECTED] 
To: [EMAIL PROTECTED] 
Subject: [obm-l] infinito 
Date: Sat, 29 Jun 2002 15:23:30 -0200 
 
Outro dia meu professor (de FISICA),fez uma 
representacao dos numeros de 0 ate + ou - 
infinito e eu nao entendi muito bem.Ele os 
colocou num circulo,e disse que nao existem 
os numeros + ou - infinito,e` somente um 
numero,e que o sinal dependeria por que lado 
voce chegaria ao infinito,igual a ideia do 
+ou- 0.Queria saber se esta certa e porque?E 
porque nao seria uma reta? 
Obrigado. 
Adriano. 
 
 
__ 
AcessoBOL, só R$ 9,90! O menor preço do mercado! 
Assine já! http://www.bol.com.br/acessobol 
 
 
= 
Instruções para entrar na lista, sair da lista e usar a lista em 
http://www.mat.puc-rio.br/~nicolau/olimp/obm-l.html 
O administrador desta lista é <[EMAIL PROTECTED]>
= 
Chat with friends online, try MSN Messenger: Click Here
=
Instruções para entrar na lista, sair da lista e usar a lista em
http://www.mat.puc-rio.br/~nicolau/olimp/obm-l.html
O administrador desta lista é <[EMAIL PROTECTED]>
=


Re: [obm-l] biquadrada...

2002-06-29 Por tôpico Marcelo Souza

Fala igor! Trankilin?
Seguinte, as raizes de um equacao briqudradas sao simetricas, naum eh? se 
sqrt3 eh raix entao -sqrt3 tb eh raiztenta isso que deve ser assim que 
sai! (e obviamente a soma destas raizes (as 4) sera zero(
valeu
abracos
marcelo!


From: Igor Castro [EMAIL PROTECTED]
Reply-To: [EMAIL PROTECTED]
To: obm-lista [EMAIL PROTECTED]
Subject: [obm-l] biquadrada...
Date: Fri, 28 Jun 2002 21:55:11 -0300

Olá colegas da lista,
podem me dar uma ajuda nesse problema?
Uma equação biquadrada de coeficientes inteiros, cuja soma desses 
coeficientes é zero, tem como uma de suas raízes sqrt(3). O produto das 
raízes dessa equação é:

eu achei 3... mas um amigo não confirmou o resultado... agradeço desde já..

[]'s Igor..



_
Chat with friends online, try MSN Messenger: http://messenger.msn.com

=
Instruções para entrar na lista, sair da lista e usar a lista em
http://www.mat.puc-rio.br/~nicolau/olimp/obm-l.html
O administrador desta lista é [EMAIL PROTECTED]
=



Re: [obm-l] Re: [obm-l] Quadriláteros

2002-06-23 Por tôpico Marcelo Souza

Como a prova disto eh geometrica (se necessario dizer, deigualdade triangular) eh possivel mostrar, se naum me engano, que a relacao
p D+d2p, a soma daas diagonais eh maior que o semiperimetro e menor que o perimetro.
Logo, p102p, logo 2p10aposto que esta questao eh de multiplla escolha, entaum procure por uma opcao que seja maior que 10, que deve se a resposta, mas achar um valor exato naum dah.
Falow
Marcelo!

From: "Igor Castro" <[EMAIL PROTECTED]>
Reply-To: [EMAIL PROTECTED] 
To: <[EMAIL PROTECTED]>
Subject: [obm-l] Re: [obm-l] Quadriláteros 
Date: Thu, 20 Jun 2002 15:36:49 -0300 
 
Fala ae rick 
Na primeira ache as duas diagonais(usando hiparco e depois ptolomeu), em 
seguida aplique a relação da mediana de euller... 
a para achar o raio, use a formula abc/4R em um triangulo que possua uma 
diagonal como lado(assim ele estará inscrito)... 
no segundo analisei apenas com existencia de triangulos e cheguei que o 
perimetro é maior que 10... mas um valor exato n achei... 
[]'s 
 
- Original Message - 
From: <[EMAIL PROTECTED]>
To: <[EMAIL PROTECTED]>
Sent: Wednesday, June 19, 2002 11:10 PM 
Subject: [obm-l] Quadriláteros 
 
 
  Olá amigos , será que poderiam me ajudar nestas duas questões : 
  1- Seja um quadrilátero inscritível ABCD cujos lados AB , BC , CD e DA 
medem 
  respectivamente 1 , 2 , 2 e 3 .Calcule a mediana de Euller do quadrilátero 
  e o raio do círculo circunscrito. 
  
  
  2-Um quadrilátero convexo O tem diagonais respectivamente iguais a 4 e 6 
  .Qual um possível valor para o seu perímetro . 
  
  Abraço.. 
  
  
   
  |-=Rick-C.R.B.=- | 
  |ICQ 124805654 | 
  |e-mail [EMAIL PROTECTED] | 
   
  
  
  -- 
  Use o melhor sistema de busca da Internet 
  Radar UOL - http://www.radaruol.com.br 
  
  
  
  = 
  Instruções para entrar na lista, sair da lista e usar a lista em 
  http://www.mat.puc-rio.br/~nicolau/olimp/obm-l.html 
  O administrador desta lista é <[EMAIL PROTECTED]>
  = 
  
 
 
= 
Instruções para entrar na lista, sair da lista e usar a lista em 
http://www.mat.puc-rio.br/~nicolau/olimp/obm-l.html 
O administrador desta lista é <[EMAIL PROTECTED]>
= 
Get your FREE download of MSN Explorer at http://explorer.msn.com.
=
Instruções para entrar na lista, sair da lista e usar a lista em
http://www.mat.puc-rio.br/~nicolau/olimp/obm-l.html
O administrador desta lista é <[EMAIL PROTECTED]>
=


Re: [obm-l] Re: [obm-l] duvidas fatoração

2002-04-25 Por tôpico Marcelo Souza

Fala cara, blz?
O fato eh que esse problema eh da eureka e eu nem li o que o garoto 
escreveu, soh li a fatoracao e lembrei na hora...pelo menos eu acho (naum 
tenho certeza) que na eureka eu li reais positivos, pus ateh no final 
escrito que x,y,z sao positivos, pois caso naum fosse estaria explicado o 
fato...mas bom vc avisar antes de eu mandar as solucoes
abracos
Marcelo


From: Rodrigo Villard Milet [EMAIL PROTECTED]
Reply-To: [EMAIL PROTECTED]
To: Obm [EMAIL PROTECTED]
Subject: [obm-l] Re: [obm-l] duvidas fatoração
Date: Wed, 24 Apr 2002 13:56:20 -0300

Acho que não é bem assim...

= (x^2+y^2-z^2)^2 -(2xy)^2 = [ (x+y)^2 -z^2 ] * [ (x-y)^2 - z^2 ]
= -(x+y+z)(x+y-z)(x-y+z)(-x+y+z) ok, até aqui tá igual. Mas vc usou o fato
de x, y, z serem positivos, o que não é dado...
Sejam P=x+y+z, Q= -x+y+z, R=x-y+z e S=x+y-z. Daí, A=-PQRS.
É fácil ver que P==Q==R==S (mod2), logo têm a mesma paridade. Se forem 
todos
ímpares, A será ímpar, portanto diferente de 2000. Então P, Q, R e S são
pares.
2000 = (2^4)*5^3 ou seja, temos 4 fatores 2 e 3 fatores 5 para distribuir
entre P,Q, R e S. Os fatores 2 já estão distribuídos. Como são 4 caras e 3
cincos, alguém fica sem nenhum 5, ou seja, é igual a +-2.
P+Q+R+S = -2(x+y+z)
As fatorações possíveis são ( a menos da ordem e do sinal de cada fator ) :
2*2*2*250, 2*2*10*50, 2*10*10*10.
Vemos que em todos os casos a soma não é múltipla de 5 ( independente dos
sinais dos fatores ), então x+y+z = +-2.
Se x+y+z = 2, A= -2(2-2z)(2-2y)(2-2x) = 16*(z-1)(y-1)(x-1). Mas agora é
fácil, pois as únicas possibilidades são 16*1*1*125, 16*1*5*25 e 16*5*5*5
  a menos da ordem e dos sinais ). Como (x-1)+(y-1)+(z-1)= -1, temos que a
terceira possibilidade é ímpossível, pois para qq escolha de sinais a soma 
é
múltipla de 5, logo não é -1. Na segunda possibilidade, temos
obrigatoriamente um fator -1, mas isso quer dizer que um dos x, y, z é 
zero.
Suponha sem perdas que seja x. Daí, A = y^4+z^4-2(yz)^2 = (y^2-z^2)^2, que 
é
quadrado, logo não pode ser 2000. E para a primeira possibilidade, é fácil
ver que a soma de 1, 1 e 125 ( podendo mudar o sinal), só pode ser 2,0,-2
mod5, logo não é -1. Então o caso x+y+z = 2 está esgotado.
Se x+y+z = -2, então A = 2(-2-2x)(-2-2y)(-2-2z) = -16(x+1)(y+1)(z+1). É bem
parecido com o caso anterior. As possibilidades são as mesmas que as do 
caso
anterior, no entanto, devemos ter obrigatoriamente álguem negativo. Assim,
esgotando todos os casos, terminamos a prova.
Ainda espero que tenha uma forma mais rápida de fazer isso, pq há um tempo
eu tinha pensado nesse problema e consegui fazê-lo mais rapidamente... qq
coisa, ou erro, avisem.
Abraços,
  Villard
-Mensagem original-
De: Marcelo Souza [EMAIL PROTECTED]
Para: [EMAIL PROTECTED] [EMAIL PROTECTED]
Data: Terça-feira, 23 de Abril de 2002 10:20
Assunto: Re: [obm-l] duvidas fatoração


 
 1.Fatore a expressão A=x^4 +y^4 +z^4 -2(x^2)(y^2) -2(y^2)(z^2) -
 -2(z^2)(x^2) e mostre que a equação A=2000 não possui solução
 inteira.
 A=(x^2-y^2-z^2)^2 -4y^2z^2
 A=(x^2-y^2-z^2-2yz)(x^2-y^2-z^2+2yz)
 A=(x^2-(y+z)^2)(x^2-(y-z)^2)
 A=(x+y+z)(x-y-z)(x+y-z)(x-y+z)
 Por desigualdade triangular, dah pra ver que este numero sempre eh
negativo,
 logo A0, para x y z positivos. Acho que eh por isso que naum tem solucao
 pra 2000
 falow.
 marcelo
 
 
 
 _
 Una-se ao maior serviço de email do mundo: o MSN Hotmail.
 http://www.hotmail.com
 
 =
 Instruções para entrar na lista, sair da lista e usar a lista em
 http://www.mat.puc-rio.br/~nicolau/olimp/obm-l.html
 O administrador desta lista é [EMAIL PROTECTED]
 =
 
 
 _
 Get your FREE download of MSN Explorer at 
http://explorer.msn.com/intl.asp.
 
 =
 Instruções para entrar na lista, sair da lista e usar a lista em
 http://www.mat.puc-rio.br/~nicolau/olimp/obm-l.html
 O administrador desta lista é [EMAIL PROTECTED]
 =
 


=
Instruções para entrar na lista, sair da lista e usar a lista em
http://www.mat.puc-rio.br/~nicolau/olimp/obm-l.html
O administrador desta lista é [EMAIL PROTECTED]
=


_
Join the world’s largest e-mail service with MSN Hotmail. 
http://www.hotmail.com

=
Instruções para entrar na lista, sair da lista e usar a lista em
http://www.mat.puc-rio.br/~nicolau/olimp/obm-l.html
O administrador desta lista é [EMAIL PROTECTED]
=



Re: [obm-l] duvidas fatoração

2002-04-23 Por tôpico Marcelo Souza


1.Fatore a expressão A=x^4 +y^4 +z^4 -2(x^2)(y^2) -2(y^2)(z^2) -
-2(z^2)(x^2) e mostre que a equação A=2000 não possui solução
inteira.
A=(x^2-y^2-z^2)^2 -4y^2z^2
A=(x^2-y^2-z^2-2yz)(x^2-y^2-z^2+2yz)
A=(x^2-(y+z)^2)(x^2-(y-z)^2)
A=(x+y+z)(x-y-z)(x+y-z)(x-y+z)
Por desigualdade triangular, dah pra ver que este numero sempre eh negativo, 
logo A0, para x y z positivos. Acho que eh por isso que naum tem solucao 
pra 2000
falow.
marcelo



_
Una-se ao maior serviço de email do mundo: o MSN Hotmail.
http://www.hotmail.com

=
Instruções para entrar na lista, sair da lista e usar a lista em
http://www.mat.puc-rio.br/~nicolau/olimp/obm-l.html
O administrador desta lista é [EMAIL PROTECTED]
=


_
Get your FREE download of MSN Explorer at http://explorer.msn.com/intl.asp.

=
Instruções para entrar na lista, sair da lista e usar a lista em
http://www.mat.puc-rio.br/~nicolau/olimp/obm-l.html
O administrador desta lista é [EMAIL PROTECTED]
=



Re: [obm-l] álgbra

2002-03-22 Por tôpico Marcelo Souza

Fala Rafael! Blz?
Seguinte, observe a identidade:
(a^2+b^2)(c^2+d^2) = (ac+-bd)^2 + (ab-+cd)^2
e use no problema para resolvê-lo, mas atenção as possibilidades (sinais)!
abraços
Marcelo


From: Rafael WC [EMAIL PROTECTED]
Reply-To: [EMAIL PROTECTED]
To: [EMAIL PROTECTED]
Subject: [obm-l] álgbra
Date: Thu, 21 Mar 2002 12:42:41 -0800 (PST)

Pessoal, já estou ficando louco com essa questão, veja
se alguém consegue resolver. Eu já teho as respostas,
mas mesmo assim não consegui uma resolução:
se (5² + 9²)(12² + 17²) for escrito sob a forma a² +
b² então a + b é igual a ?
resp: 236 ou 286 (213² + 23² ou 193² + 93²)

Obrigado,

Rafael.

=
Rafael Werneck Cinoto
   ICQ# 107011599
 [EMAIL PROTECTED]
   [EMAIL PROTECTED]
   [EMAIL PROTECTED]
http://www.rwcinoto.hpg.com.br/

__
Do You Yahoo!?
Yahoo! Movies - coverage of the 74th Academy Awards®
http://movies.yahoo.com/
=
Instruções para entrar na lista, sair da lista e usar a lista em
http://www.mat.puc-rio.br/~nicolau/olimp/obm-l.html
O administrador desta lista é [EMAIL PROTECTED]
=


_
Get your FREE download of MSN Explorer at http://explorer.msn.com/intl.asp.

=
Instruções para entrar na lista, sair da lista e usar a lista em
http://www.mat.puc-rio.br/~nicolau/olimp/obm-l.html
O administrador desta lista é [EMAIL PROTECTED]
=



Re: [obm-l] determinantes

2002-03-17 Por tôpico Marcelo Souza

Tipo, acho que o que vc quer dizer eh sobre partições de matrizes (não sei 
bem se o nome eh esse pq estudei num livro em frances e não sei mto 
frances). Seja uma matriz A. Decompor A em matrizes de ordens inferiores, 
vejamos:
a_11 a_12|a_13
A= _a_21_a_22|a_23_
a_31 a_32|a_33
os blocos são constituídos das seguintes matrizes:
P= a_11 a_12Q= a_13R=a_31 a_32   S = a_33
   a_21 a_22   a_23
Escrevemos

A = P  Q
R  S
Nisto vc dividiu a matriz em blocos.
A regra que eu conheço eh assim:

   A_1
  A_2
A=   ...
A_s
entenda A como uma matriz e tome A_i (i=1,2,...,s) como blocos de matrizes 
quadradas (chamadas quase diagonais).
Eu sei que det A = detA_1...detA_s.
Mas não sei dizer se o que vc falou está certo, procurei no livro e não 
consegui ver coisa alguma. As vezes eu olhei rapido demais... mas vai aí o 
toque. OK?
Abraços
Marcelo
From: Siberia Olympia [EMAIL PROTECTED]
Reply-To: [EMAIL PROTECTED]
To: [EMAIL PROTECTED]
Subject: [obm-l] determinantes
Date: Sun, 17 Mar 2002 15:06:41 -0300

 Se X é uma matriz 2n x 2n que é dividida em quatro blocos (matrizes) n 
x
n, a saber, A , B, C e D (Estou supondo que em cima ficam os blocos A e B e
em baixo ficam os blocos C e D, nesta ordem). É verdade que
detX=detA.detD-detC.detB ?

cgomes

=
Instruções para entrar na lista, sair da lista e usar a lista em
http://www.mat.puc-rio.br/~nicolau/olimp/obm-l.html
O administrador desta lista é [EMAIL PROTECTED]
=


_
Send and receive Hotmail on your mobile device: http://mobile.msn.com

=
Instruções para entrar na lista, sair da lista e usar a lista em
http://www.mat.puc-rio.br/~nicolau/olimp/obm-l.html
O administrador desta lista é [EMAIL PROTECTED]
=



Re: [obm-l] determinantes

2002-03-17 Por tôpico Marcelo Souza

Oi Morgado, td bom?
Estranho...
O que eu vi do livro (elements du calcul numerique-MIR), que coloquei no 
email anterior, não dah resposta certa pro determinante exemplo que vc 
deu...o que há de errado? (digo, coloco talvez a culp+a em mim pelo fato de 
eu não saber frances)
abraços
Marcelo



From: Augusto César Morgado [EMAIL PROTECTED]
Reply-To: [EMAIL PROTECTED]
To: [EMAIL PROTECTED]
Subject: Re: [obm-l] determinantes
Date: Sun, 17 Mar 2002 19:37:29 -0300

eh falso.
Considere
1 2 5 6
3 4 7 8
0 3 4 1
1 2 9 2
Se fosse verdadeiro, a resposta seria menos 4.
O determinante vale menos 176.


Siberia Olympia wrote:

Se X é uma matriz 2n x 2n que é dividida em quatro blocos (matrizes) n 
x
n, a saber, A , B, C e D (Estou supondo que em cima ficam os blocos A e B 
e
em baixo ficam os blocos C e D, nesta ordem). É verdade que
detX=detA.detD-detC.detB ?

cgomes

=
Instruções para entrar na lista, sair da lista e usar a lista em
http://www.mat.puc-rio.br/~nicolau/olimp/obm-l.html
O administrador desta lista é [EMAIL PROTECTED]
=




=
Instruções para entrar na lista, sair da lista e usar a lista em
http://www.mat.puc-rio.br/~nicolau/olimp/obm-l.html
O administrador desta lista é [EMAIL PROTECTED]
=


_
Send and receive Hotmail on your mobile device: http://mobile.msn.com

=
Instruções para entrar na lista, sair da lista e usar a lista em
http://www.mat.puc-rio.br/~nicolau/olimp/obm-l.html
O administrador desta lista é [EMAIL PROTECTED]
=



Re: [obm-l] Duas perguntinhas

2002-02-20 Por tôpico Marcelo Souza

Olá Daniel,
a primeira eh bastante simples, não é que não existe nobel d4e matemática, 
eh que esse nobel tem nome diferente, o nobel da matemática eh a medalha 
fields, dada de 4 em 4 anos...se não me engano.
Eu fico devendo a hipótese de riemman pq eu a li faz tempo e não consigo 
lembrar e não eh uma coisa que eu estou sempre vasculhando...
ok!
abraços
Marcelo



From: Daniel [EMAIL PROTECTED]
Reply-To: [EMAIL PROTECTED]
To: Lista da OBM [EMAIL PROTECTED]
Subject: [obm-l] Duas perguntinhas
Date: Tue, 19 Feb 2002 21:46:10 -0300

Olá à todos, uma pergunta que nunca me souberam responder, porque não 
existe prêmio nobel de Matemática?
 A segunda é relativa a Hipótese de Rieman, do que ela 
trata de forma resumida, se é que isto é possível? Acho que inclusive já 
foi discutido na lista, porém perdi tal discussão.

Obrigago a todos



_
Chat with friends online, try MSN Messenger: http://messenger.msn.com

=
Instruções para entrar na lista, sair da lista e usar a lista em
http://www.mat.puc-rio.br/~nicolau/olimp/obm-l.html
O administrador desta lista é [EMAIL PROTECTED]
=



Re: [obm-l] Re: [obm-l] Re:

2002-02-12 Por tôpico Marcelo Souza


Eu acho que a resposta eh 10 km/h
abraos
Marcelo

From: [EMAIL PROTECTED] 
Reply-To: [EMAIL PROTECTED] 
To: [EMAIL PROTECTED] 
Subject: [obm-l] Re: [obm-l] Re: 
Date: Mon, 11 Feb 2002 20:51:57 -0300 
 
Ola. Mais uma pra vcs brincarem: 
 
 
Tres turistas, que possuem uma bicicleta, devem chegar ao centro turistico 
no menor espaco de tempo (o tempo conta-se ate que o ultimo turista chegue 
ao centro), 
A bicicleta pode transportar apenas duas pessoas e, por isso, o terceiro 
turista deve ir a pe. O 1o turista assume-se como ciclista. Este leva o 
segundo turista ate um determinado ponto do caminho de onde este continua 
a andar a pe e o ciclista regressa para transportar o terceiro. Encontrar 
a velocidade media dos turistas, sendo a velocidade de quem vai a pe 4 km/h 
e de quem vai na bicicleta 20 km/h.  
 
Abracos. 
 
Asselin. 
-- Mensagem original -- 
 
 Sendo esta uma desigualdade simtrica, assuma x=y=z eleve ao cubo ambos 
 os 
 lados, passe tudo para o lado esquerdo e rearranje de forma que as soma 
das 
  
 parcelas, de acordo com as diferenas e produtos dados sejam sempre 
 positivos...esta eh uma maneira um pouco suicida de se fazer j que eh 
pouco 
  
 criativa. 
 abraos 
 Marcelo 
  
  
 From: [EMAIL PROTECTED] 
 Reply-To: [EMAIL PROTECTED] 
 To: [EMAIL PROTECTED] 
 Date: Mon, 11 Feb 2002 11:16:57 -0300 
  
 Sejam x, y, z reais positivos. Prove: 
  
 (x+y+z)/3 = 3rd root de (xyz) 
  
 Depois generalize para n reais. 
 O caso para n=2 eh o mais simples. 
 Como provar sem se basear neste caso? 
 Alguem usaria o Polinomio de Leibniz ? 
  
 Abracos, 
 Asselin. 
  
  
  
 -- 
 Use o melhor sistema de busca da Internet 
 Radar UOL - http://www.radaruol.com.br 
  
  
  
 = 
 Instrues para entrar na lista, sair da lista e usar a lista em 
 http://www.mat.puc-rio.br/~nicolau/olimp/obm-l.html 
 O administrador desta lista <[EMAIL PROTECTED]> 
 = 
  
  
 _ 
 Send and receive Hotmail on your mobile device: http://mobile.msn.com 
  
 = 
 Instrues para entrar na lista, sair da lista e usar a lista em 
 http://www.mat.puc-rio.br/~nicolau/olimp/obm-l.html 
 O administrador desta lista <[EMAIL PROTECTED]> 
 = 
  
 
 
 
-- 
Use o melhor sistema de busca da Internet 
Radar UOL - http://www.radaruol.com.br 
 
 
 
= 
Instrues para entrar na lista, sair da lista e usar a lista em 
http://www.mat.puc-rio.br/~nicolau/olimp/obm-l.html 
O administrador desta lista <[EMAIL PROTECTED]> 
= 
Get your FREE download of MSN Explorer at http://explorer.msn.com.
=
Instruções para entrar na lista, sair da lista e usar a lista em
http://www.mat.puc-rio.br/~nicolau/olimp/obm-l.html
O administrador desta lista é <[EMAIL PROTECTED]>
=


[obm-l] Problemas afinal!!!! =)

2002-02-10 Por tôpico Marcelo Souza

Essa lista está ficando muito monótona, sem muitas discussões sobre 
problemas, só o pessoal atacando na teoria. Vou colocar alguns problemas 
aqui e espero que vocês mandem soluções =)
1. Dada a sequencia infinita de inteiros a_1,a_2,..., definida por
a_1 = 1, a_2=0,a_3=-5 e a_n=4[a_(n-1)]-5[a_(n-2)]+2[a_(n-3)]   n=3
ache uma expressão fechada para a_n.
2. Prove a seguinte desigualdade:
x,y,z reais positivos, para r0
[x^r](x-y)(x-z)+[y^r](y-x)(y-z)+[z^r](z-x)(z-y)=0
Com igualdade x=y=z, ou então se dois deles forem iguais e o terceiro igual 
a 0.
3.Sejam a,b,c reais positivos satisfazendo abc=1. Mostre que:
1/a^3(b+c) + 1/b^3(a+c) + 1/c^3(a+b)=3/2
valeu
abraços
Marcelo

_
MSN Photos is the easiest way to share and print your photos: 
http://photos.msn.com/support/worldwide.aspx

=
Instruções para entrar na lista, sair da lista e usar a lista em
http://www.mat.puc-rio.br/~nicolau/olimp/obm-l.html
O administrador desta lista é [EMAIL PROTECTED]
=



Re: triângulo

2002-01-11 Por tôpico Marcelo Souza

A primeira, parece não ser tão dif~icil...se vc fizer lei dos senos, e usar 
trigonometria dah para vc obter relações simples..
a segunda parece um jogo de substituições...uma exponencial...do tipo.
Depois eu mando a solução com mais calma =)
abraços
Marcelo
From: Eder [EMAIL PROTECTED]
Reply-To: [EMAIL PROTECTED]
To: [EMAIL PROTECTED]
Subject: triângulo
Date: Sun, 6 Jan 2002 20:52:13 -0200

No triângulo ABC  C=3A (ângulos),a=27 e c=48.Quanto mede b?

Até agora não resolvi essa...

Outra:

Se (a^b)=(b^a) e b=(9^a),qual o valor de a?

Conto com a ajuda dos colegas de lista.


_
Join the world’s largest e-mail service with MSN Hotmail. 
http://www.hotmail.com




Re: Onde compro esses livros?

2001-12-30 Por tôpico Marcelo Souza

Olha, esses livros, naum to querendo ser chato, mas custam baratissimo, para 
naum dizer o contrario. Pois eu vou mais pela editora que sei que é 
Springer'Verlag, se nao me engano...A media desses livros na epoca da naum 
alta do dolar era por volta de 100 reais )o mais baratoum cara da 
livraria me disse uma vez que naum valia a pena comprar pois quase naum saia 
por causa dos preços altos. Devem ser muito caros aposto eu e por isso naum 
se deve encontarar aqui no brasil pra vender assim tao facilm, ainda mais 
com a alta do dolar.
abraços
Marcelo


From: Jorge Peixoto de Morais Neto [EMAIL PROTECTED]
Reply-To: [EMAIL PROTECTED]
To: [EMAIL PROTECTED]
Subject: Onde compro esses livros?
Date: Sun, 30 Dec 2001 16:38:18 -0200

Eu estava procurando os livros Problem-Solving Strategies, Winning 
Solutions e um outro que era uma compilação de IMO's. Só achei no 
Amazon.com e outras livrarias norte-americanas; obviamente, é caríssimo e
demora muito para chegar. Alguem sabe em que outro lugar (de preferência no 
Brasil, claro) eu posso achar esses livros ?

[]s, Jorge Peixoto



_
Join the world’s largest e-mail service with MSN Hotmail. 
http://www.hotmail.com




Re: beal

2001-12-16 Por tôpico Marcelo Souza

2) Vc quer aprender indução, é isso? Eu acho que o artigo do Elon da revista 
Eureka é uma boa pedida para um treino assim como para um aprendizado, está 
bem explicado, não está confuso...É bom ler, mas é melhor ainda ter certeza 
do que se pode fazer com indução.
O princípio da indução diz, basicamente, que, dada uma propriedade S(n) 
válida para um número n natural. Se S(1) é válida e, se o fato de S(K) valer 
implicar que S(K+1) vale, então, S vale para todos os naturais.
Vejamos um exemplo simples:
Mostre que 1+2+3+...+n = [n(n+1)]/2
Primeiro passo: Ver se vale para n=1
1=1(2)/2 =1 (0K)
Segundo: Assuma que vale para K e tente provar para K+1
Se vale para K então

1+2+...+k = k(k+1)/2
Vc quer provar para k+1, certo? Logo, o lado esquerdo está precisando de 
somar k+1, para não alterar, somar dos dois lados
1+2+...+k+(k+1)=k(k+1)/2 + (k+1)
   = (k+1)(k+2)/2
Isto prova que vale para k+1, pois note que é a mesma fórmula de k, mas com 
k+1 ao invés de k.
Faça como exercício esta
Mostrar que 1+2^2+3^2+...+n^2=n(n+1)(2n+1)/6
Ok
valeu
Marcelo


From: gabriel guedes [EMAIL PROTECTED]
Reply-To: [EMAIL PROTECTED]
To: [EMAIL PROTECTED]
Subject: beal
Date: Sat, 15 Dec 2001 18:46:37 -0200

tudo bem colegas da lista,
1)Alguem ja ouviu  falar na conjectura de beal oque que ela propõe e etc???

2)Estava dando uma olhada em indução finita , e queria me a profundar 
,alguem conhece um bom livro  ?


_
MSN Photos is the easiest way to share and print your photos: 
http://photos.msn.com/support/worldwide.aspx




Re: Potência infinita ?

2001-12-09 Por tôpico Marcelo Souza


Ultimamente ando com tempo para responder mensagens...hehehevantagens de frias
Mas vamos l:
x^x^...=2
Esquece o primeiro x. A diante teramos x^x^... de novo e isso  igual a 2. Logo, aquilo lah em cima  o mesmo que x^2=2 = x=sqrt2
abraos
Marcelohehe...ultimamente ando com tempo para respo 

From: [EMAIL PROTECTED] 
Reply-To: [EMAIL PROTECTED] 
To:<[EMAIL PROTECTED]> 
Subject: Potncia infinita ? 
Date: Sun, 09 Dec 2001 01:49:34 EST 
 
 Ol colegas da lista, 
 Vi uma resoluo de uma interessante questo com potncias que muito depois j no me parece correta. Quero saber se h algum erro. Obrigado. 
 Trata-se de uma incgnita que est eleva a ela mesma infinitas vezes (sem parnteses) igual a dois, isto , x elevado a x, que este est elevado a x ... igual a dois. A soluo vem da percepo de que pode-se esquecer do primeiro x (da base) e substituir o resto por dois. Tm se assim x ao quadrado igual a dois. At a h algum erro ? O x pode valer + ou - a raiz de dois ? 
 O que me faz parecer que h um erro  que se mudarmos o problema trocando o dois por quatro, a soluo permanece a mesma. Assim essa potncia infinita seria algo indeterminado pois pode valer dois ou quatro. 
 Obrigado pela ateno. 
 Raul 
Get your FREE download of MSN Explorer at http://explorer.msn.com


Re: Potência infinita ?

2001-12-09 Por tôpico Marcelo Souza

Desculpa, não tinha lido a essencia da sua pergunta.

vejamos x^x^...=k = x^k=k = x=raiz k-ésima de k
Se vc mudar dois por quatro, vc na realidade, não mudou absolutamente nada, 
visto que sqrt2 é o mesmo que raiz quarta de 4. Teoricamente, se vc ir 
substituindo k por números 3,4,5,6...cada vez terá um número menor para x, 
pois a sequencia 1, sqrt2,raiz cubica de 3,
raiz quarta de 4...e assim por diante é decrescente a partir do terceiro 
termo. Isto é fácil de se provar por indução. Há um equivoco em dizer que a 
potencia infinita pode valer 2 ou 4...A potencia infinita pode valer quanto 
o cara que montou o problema quiser  . Depende do que há do outro lado da 
igualdade. Isso determina o valor da potencia infinita. O fato de ser 2 ou 4 
implica que valem, na realidade a mesma coisa o valor de x. O que há com o 
seu raciocinio, pelo menos é o que eu estou achando, é uma confusão de 
incognitas.
se eu disser
x^x^...=2 e x^x^...=4 aí é um absurdo, pois estou afirmando que 2=4...Vc 
deve estar confundindo as incognitas.
A resposta disso é sempre x=raiz k-esima de k para x^x^x^...=k
ok?
Qquer equivoco, me perdoe
Um abraço
Marcelo
From: [EMAIL PROTECTED]
Reply-To: [EMAIL PROTECTED]
To: [EMAIL PROTECTED]
Subject: Potência infinita ?
Date: Sun, 09 Dec 2001 01:49:34 EST

   Olá colegas da lista,
   Vi uma resolução de uma interessante questão com potências que muito 
depois já não me parece correta. Quero saber se há algum erro. Obrigado.
   Trata-se de uma incógnita que está eleva a ela mesma infinitas vezes 
(sem parênteses) igual a dois, isto é, x elevado a x, que este está elevado 
a x ... igual a dois. A solução vem da percepção de que pode-se esquecer do 
primeiro x (da base) e substituir o resto por dois. Têm se assim x ao 
quadrado igual a dois. Até aí há algum erro ? O x pode valer + ou - a raiz 
de dois ?
   O que me faz parecer que há um erro é que se mudarmos o problema 
trocando o dois por quatro, a solução permanece a mesma. Assim essa 
potência infinita seria algo indeterminado pois pode valer dois ou 
quatro.
   Obrigado pela atenção.
  Raul


_
Get your FREE download of MSN Explorer at http://explorer.msn.com/intl.asp




Re: Teoria dos números

2001-12-09 Por tôpico Marcelo Souza

Fala Raul, blzura?
Seguinte, a RPM está no volume 46, se não me engano.É um pouquinho difícil 
consegui-la. Mas creio que mandando um  email ou carta para o ime-usp vc 
consigaespero que alguém coloque o endereço.
É verdade, pelo teorema de fermat seria muito mais simples resolveer a 
questão, mas observe: MUITO MAIS SIMPLES!
tendo K^5 - K vc pode fatorar
K(K^2+1)(K+1)(K-1)
daí é so montar uma tabela de valores para K
Vc tem que fixar no ultimo algarismo de K, pois ele vai definir tudo. Tente 
K=0,1,...,9 que são todos os algarismos e observe que funciona.
É fácil ver que é divisível sempre por dois (pois tem dois caras 
consecutivos na história). Pra provar que é por cinco é só fazer o que eu 
falei acima.
corrijam se eu estive4 errado
abraços
Marcelo

From: [EMAIL PROTECTED]
Reply-To: [EMAIL PROTECTED]
To: [EMAIL PROTECTED]
Subject: Teoria dos números
Date: Sun, 09 Dec 2001 11:41:16 EST

   Olá colegas,
   obrigado pela atenção na questão de potências e, relativo a ela, onde 
encontro a RPM 26 ?
   Agora, teve uma questão do IME que um aluno me mostrou e só sei resolver 
usando o pequeno teorema de Fermat, gostaria de saber se há outra 
resolução.
   Trata-se de provar que K e K^5 terminam com o mesmo algarismo para todo 
K inteiro.
   Prova-se usando que K^5 - K é divisível por dois e por 5 (onde se usa o 
pequeno teorema).
   Acho que pode haver outra resolução pois não acredito que o IME queria 
cobrar o pequeno teorema de Fermat, ou será possível ?
   Obrigado pela atenção,
  Raul


_
Get your FREE download of MSN Explorer at http://explorer.msn.com/intl.asp




Torneios das Cidades

2001-12-09 Por tôpico Marcelo Souza

Como está cotado ultimamente e essa área de empréstimo de problemas está bem 
legal, quero dizer que tenho aqui problemas dos torneios das Cidades, desde 
a décima sétima competição até a vigesima segunda. Quem estiver interessado, 
pode pedir para mim pelo meu email pessoal, ok?
Abraços
Marcelo

_
Get your FREE download of MSN Explorer at http://explorer.msn.com/intl.asp




Re: Fazendo contas rapidas

2001-12-08 Por tôpico Marcelo Souza

Oi, eu onhecço um legal . Para elevar números terminados em 5 ao quadrado 
basta

35^2

  3   5
X4
12   25
ou seja 1225
Separa o 5 e multiplica os anteriores pelo seu consecutivo
, depois coloca no final do numero 25. exemplo 45

separa o cinco,,,fica 4X5 = 20coloca o 25 no final 2025
isso é fácil de se oservar. Vamos escrever um numero n como
n=10^kx_k + ... +10x_1 + 5
colocando o 10 em evidência e chamando o que ficou no parenteses de m, temos
n=10m+5 = n^2 = 100m^2+100m+25 = 100m(m+1)+25
tah exlpicado aí, pq isso acontece. Isso facilita as contas ateh certo 
ponto, tente fazer 2233445^2 =)
abraços
Marcelo
From: Gustavo Nunes Martins [EMAIL PROTECTED]
Reply-To: [EMAIL PROTECTED]
To: obm-l [EMAIL PROTECTED]
Subject: Fazendo contas rapidas
Date: Fri, 07 Dec 2001 15:30:12 -0200

Alguem pode me contar (ou me dizer onde posso achar) alguns truques
para fazer algumas contas de modo rapido?

Por exemplo, fazer 45x53 de cabeca e facil e rapido pois fazemos 2500  -
15 - 100
Podemos ver isso assim: (a+b)(a+c) = a^2 + (b+c)a + bc. No nosso caso,
a=50, b=-5 c=3

Obrigado,
Gustavo



_
Get your FREE download of MSN Explorer at http://explorer.msn.com/intl.asp




Re: 9997-urgente!

2001-11-30 Por tôpico Marcelo Souza

Eu fiz de um modo bem,digamos, infantil... Escrevi 9997m = 1000m - 3m
m=x_0x_1...x_n

(x_n...x_1x_0)
-  ...x_1'x_0'
onde esses x_i' são os algarismos da unidade qdo multiplicamos x_i por 
3completei de trás pra frente, escolhendo os menores
Achei 9995trabalhosamentee duvidosamentequeria um método 
mais preciso, se possível...
abraços
M.


From: [EMAIL PROTECTED]
Reply-To: [EMAIL PROTECTED]
To: [EMAIL PROTECTED]
Subject: Re: 9997-urgente!
Date: Thu, 29 Nov 2001 14:38:10 -0300

Oi Marcelo,
k = (3335)*9997 = 9995, como você vê, funciona. Creio que seja o
menor, mas não tenho certeza. Vou escrever o que pensei, faça(você ou outra
pessoa da lista que entenda de teoria dos números melhor que eu) um 
julgamento
crítico:
Um múltiplo de 9997 pode ser escrito como: 1*n - 3*n.
É claro que n par não serve. Vejamos então n ímpar.
O número 1*n é escrito como n1n2...nm, em que n1n2...nm são os
dígitos de n. Quando fizermos 1*n - 3*n, teremos que pegar 1 
emprestado
de nm. Mas n é ímpar, logo nm é ímpar. Se n = , 3*n  1. Então
o quinto dígito(da direita pra esquerda) do múltiplo será par(ele será 
simplesmente
nm - 1).
Testando o próximo n ímpar, encontrei a resposta acima.

abraço
   Camilo



--
Use o melhor sistema de busca da Internet
Radar UOL - http://www.radaruol.com.br





_
Get your FREE download of MSN Explorer at http://explorer.msn.com/intl.asp




Re: Binomial :)

2001-11-29 Por tôpico Marcelo Souza

é isso simeu pereguntei pra dividir somente pois estava errando as 
contas diretotava dando uma eq do quarto grau...depois que eu me liguei 
que estava errando num detalhe.
abraços
M.


From: Rodrigo Villard Milet [EMAIL PROTECTED]
Reply-To: [EMAIL PROTECTED]
To: [EMAIL PROTECTED]
Subject: Re: Binomial :)
Date: Wed, 28 Nov 2001 22:53:47 -0200

Tente quebrar as expressões... daí, 1/(k-1)!*(n-k+1)!  + 1/(k+1)!*(n-k-1)! 
=
2/k!*(n-k)!, ou seja, 1/(n-k)*(n-k+1) + 1/k(k+1) = 2/k*(n-k) que é
equivalente a : k(k+1) + (n-k)*(n-k+1) = 2*(k+1)*(n-k+1)...
k^2+k+n^2-nk+n-nk+k^2-k=2nk-2k^2+2k+2n-2k+2 ... 4k^2  + n^2 - 4nk  - n -2=0
... (2k-n)^2 = n+2, ou seja, temos que k = [n + sqrt(n+2)]/2. No entanto k 
é
inteiro, então n = q^2 - 2 e assim , k = (q^2+q-2)/2. E ainda temos que ter
kn, logo (q^2+q-2)/2  q^2 - 2 ... (q^2 - q -2)/20 , logo, q =3.
Será que valem todos os n da forma q^2 - 2, com q natural =2 ?? Hum...
Testei pra q = 3 e q = 4, valeu sim... basta provar por indução em q eu 
acho
:)). Resposta : n = q^2 - 2 , onde q é natural =3. :)
Abraços, Villard !

Comentem, por favor...


-Mensagem original-
De: Marcelo Souza [EMAIL PROTECTED]
Para: [EMAIL PROTECTED] [EMAIL PROTECTED]
Data: Terça-feira, 27 de Novembro de 2001 19:23


 Por favor, alguém poderia me dizer qto vale
 
 ( n ) ( n )( n )   ( n )
 (   )+(   ) = 2(   ) dividindo todo mundo por  (   )
 (k-1) (k+1)( k )   ( k )
 
 onde
 ( n )
 (   ) = n!/k!(n-k)!
 ( k )
 obrigado
 abraços
 
 
 _
 Get your FREE download of MSN Explorer at 
http://explorer.msn.com/intl.asp
 



_
Get your FREE download of MSN Explorer at http://explorer.msn.com/intl.asp




9997-urgente!

2001-11-29 Por tôpico Marcelo Souza
Como faço para encontrar o menor múltiplo de 9997, que não seja 9997, e tenha todos os dígitos ímpares?
abraços
M.Get your FREE download of MSN Explorer at http://explorer.msn.com


Re: 2 de geometria

2001-11-28 Por tôpico Marcelo Souza

A primeira parece um problema da olimpiada estadual do RJ antiga..mas acho 
que ela pedia para determinar o comprimento da paralela compreendida entre 
os lados. É fácil ver que os três triangulos formados são semelhantes ao 
inicial...acho que sai por aí...vou entar.
abraços
M.


From: Alexandre Tessarollo [EMAIL PROTECTED]
Reply-To: [EMAIL PROTECTED]
To: [EMAIL PROTECTED]
Subject: Re: 2 de geometria
Date: Wed, 28 Nov 2001 01:50:14 -0200

Aqueles que quiserem uma figurinha, podem me pedir, eu tenho aqui. Pela
recente mesngame do volume de óleo já vi q é possível mandar anexos, mas
antes gostaria de saber qual a política oficial da lista com respeito a
anexos. Nicolau, por favor...

Alexandre Tessarollo wrote:
 
  Ao povo q gosta de gemetria, seguem duas. A primeira foi um 
aluno que
  me passou, tirada de um livro de segundo grau das antigas (Exercícios
  de Gemetria Plana, do Edgar de Alencar Filho). A outra acho q até já
  postei aqui, não tenho certeza. Mas tirei de outra lista. Vamos a elas:
 
  1) Por um ponto P interno a um triângulo ABC traçam-se a rretas
  paralelas aos seus lados, que o decompõem em seis partes, três das quais
  triângulos de área S[1], S[2] e S[3]. Achar a área do triângulo ABC.
 
  Eu até tenho a respoata, mas ainda não sei como chegar nela.
 
  2) Tome um pentágomno qualquer ABCDE e as retas suportes dos lados. Note
  que elas detrminam um triângulo em cima de cada lado do pentágono.
  Construa as circunferências circunscritas a esse triângulos. Note que as
  circunferências de lados adjacentes se interceptam duas a duas em dois
  pontos: um dos vértices do pentágono e outro. Chamemos esses outros
  pontos convenietemente de A', B', C', D' e E'. Prove que A', B', C', D'
  e E' pertencem a uma mesma circunferência.
 
  Não lembro de qual das n! listas que tirei esta questão, mas 
lembro q
  mencionava uma relação não confirmada a algum político chinês recente.
  Não sei se teria sido este chinês a formular ou se foi devido a ele que
  o prob ficou conhecido. Seria uma caso similar ao problema do cavalo do
  presidente, né Nicolau? :0)))
 
  []'s
 
  Alexandre Tessarollo


_
Get your FREE download of MSN Explorer at http://explorer.msn.com/intl.asp




[no subject]

2001-11-27 Por tôpico Marcelo Souza

Por favor, alguém poderia me dizer qto vale

( n ) ( n )( n )   ( n )
(   )+(   ) = 2(   ) dividindo todo mundo por  (   )
(k-1) (k+1)( k )   ( k )

onde
( n )
(   ) = n!/k!(n-k)!
( k )
obrigado
abraços


_
Get your FREE download of MSN Explorer at http://explorer.msn.com/intl.asp




binomial

2001-11-27 Por tôpico Marcelo Souza

Por favor, algum poderia me dizer qto vale 
( n ) ( n ) ( n ) ( n ) 
( )+( ) = 2( ) dividindo todo mundo por ( ) 
(k-1) (k+1) ( k ) ( k ) 

onde 

( n ) 
( ) = n!/k!(n-k)! 
( k ) 
obrigado 
abraos 
Get your FREE download of MSN Explorer at http://explorer.msn.com


Re: Postulado de Bertrands e Complexos

2001-11-03 Por tôpico Marcelo Souza

2)postulado de bertrand: Cara, o troco naum e mto breve, se vc quiser, 
depois mando um completo pelo pessoal (com adicao de Lemas e teoremas)
blz
[]'s, M.
From: Jose Paulo Carneiro [EMAIL PROTECTED]
Reply-To: [EMAIL PROTECTED]
To: [EMAIL PROTECTED]
Subject: Re: Postulado de Bertrands e Complexos
Date: Fri, 2 Nov 2001 22:41:25 -0200

1) Dois complexos (nao nulos) z e w estao alinhados com a origem se e so 
se:
z/w eh real;
z/w eh o seu proprio conjugado;
zw' =z'w  (aqui z' eh o conjugado de z)

2) Consequentemente, os complexos z, w, u estao alinhados se e so se:
(z-w)(u'-w') = (z'-w')(u-w)
Esta condicao eh equivalente a nulidade do determinante cujas linhas sao:
1, w, w'
1, z, z'
1, u, u'

3) Os complexos u,v,w,z sao cociclicos se e so se:
o angulo zu,zv (isto eh, a rotacao que leva o unitario de zu a coincidir 
com o unitario de zv)eh o mesmo ou eh o suplemento do angulo wu,wv (faca 
uma figura: as 2 possibilidades correspondem aos casos em que z e w estao 
no mesmo arco determinado por u e v ou em arcos replementares).
Isto significa que u-z / v-z eh um multiplo real (positivo no 1o caso, e 
negativo no 2o caso)de u-w / v-w, isto eh:
(u-v)(v-w)/(v-z)(u-w) eh real.
Este quociente se chama razao cruzada ou razao dupla.

JP

   - Original Message -
   From: Marcos Eike
   To: [EMAIL PROTECTED]
   Sent: Friday, November 02, 2001 8:38 PM
   Subject: Postulado de Bertrands e Complexos


   primeiro:

   Alguém conhece alguma prova para o seguinte teorema.

   Para n inteiro maior que 1, há pelo menos um primo p tal que n  p  2n

   segundo:

   Como provar que existe pontos colineares e conciclicos usando números 
complexos?

   Um problema que tem no artigo de números complexos da revista Eureka, 
porém não conseguir entender a solução.
   Quem puder tecer alguns comentários, eu agradeceria. ( o meu mair medo 
de aplicar os complexos em geometria é a visão cartesiana que tenho de 
procurar algum eixo ou ponto de referência)

   Problema:
   Seja ABC um triângulo, H o seu ortocentro, O o seu circuncentro e R o 
seu circunraio. Seja D o simétrico de A com relação a BC, E o simétrico de 
B com relação a AC e F o simétrico de C com relação a AB.

   Prove que D, E e F são colineares se, e somente se, OH = 2R.





   Ats,
   Marcos Eike



_
Get your FREE download of MSN Explorer at http://explorer.msn.com/intl.asp




Re: Geometria - interessantes

2001-11-01 Por tôpico Marcelo Souza

São problemas correlatos. O primeiro é da iberoamericana (so que acho que os 
valores eram diferentes)...e o segunda da olimpiada estadual do Rio.
1.basta fazer a rotação do triangulo APB por exemplo (qquer um serve) de 
60º. Dai, ficamos com um triangulo equilatero, no caso 7,7,7. e um triangulo 
7,8,5. Calculamos o cosseno de CP'P, onde P' é a rotação do ponto P. Esse 
cosseno vale 1/7. (usando lei dos cossenos). Depois usando lei dos cossenos 
no triangulo CP'B, acha-se o lado do triangulo.
2. Em baixo o mesmo esquema. Use a diagonal para formar um triangulo e use o 
mesmo processo acima.
Desculpe a falta de clareza, mas sem desenho e dificil.
[]'s, M


From: Guilherme Pimentel [EMAIL PROTECTED]
Reply-To: [EMAIL PROTECTED]
To: Lista OBM [EMAIL PROTECTED]
Subject: Geometria - interessantes
Date: Thu, 1 Nov 2001 04:57:09 -0200

Tirei estes aqui de outra lista, achei interessantes:

[1] Seja ABC um triangulo equilatero e P um ponto interior distando 5, 7 e 
8
dos vertices. Ache o lado do triangulo.

[2] Seja ABCD um quadrado e P um ponto interior que dista 1 de A, 4 de B e 
5
de C. Ache a area do quadrado.

[]'s Guilherme Pimentel
http://sites.uol.com.br/guigous
 winmail.dat 


_
Get your FREE download of MSN Explorer at http://explorer.msn.com/intl.asp




RE: Quadrados perfeitos...

2001-10-29 Por tôpico Marcelo Souza

O fato de saber que um quadrado perfeito so pode ser da forma 4k ou 4k+1 tb 
ajudaembora somente saber a observacao do eduardo conclua o problema.
[]'s M.


From: Eduardo Grasser [EMAIL PROTECTED]
Reply-To: [EMAIL PROTECTED]
To: '[EMAIL PROTECTED]' [EMAIL PROTECTED]
Subject: RE: Quadrados perfeitos...
Date: Mon, 29 Oct 2001 10:26:54 -0200

ajuda saber que quadrados perfeitos terminam em 0, 1, 4, 5, 6, 9? Fatore só 
os que terminarem nestes números...

Eduardo Grasser

--
De:Fernando Henrique Ferraz[SMTP:[EMAIL PROTECTED]]
Enviada em:Sábado, 27 de Outubro de 2001 15:26
Para:  [EMAIL PROTECTED]
Assunto:   Quadrados perfeitos...

Vi esse exercício numa prova de vestibular desse ano,

28. Qual dos números seguintes é quadrado perfeito?

a) 745328
b) 9015743
c) 6259832
d) 9761387
e) 14641

O jeito mais óbvio parece fatorar um a um.. mas é muito braçal e leva muito
tempo. Existe alguma regra que indique se o número é quadrado perfeito ou 
não?
Um amigo sugeriu que a soma dos algarismos que compõe um quadrado perfeito
dá outro quadrado perfeito... mas nem sempre é válida...
funciona para 121 ... 1 + 1 + 2 = 4
Mas não para 256.. = 13
(curiosamente dá certo no 14641)





Against stupidity, the Gods themselves contend in vain,
  Friedrich von Schiller's
-
[]'s
{O-Grande-Mentecapto}
[EMAIL PROTECTED]




Procuradoria Regional do Trabalho da 15ª Região - Campinas/SP

***

Por medida de segurança, os arquivos com extensões: .exe .vb[es]
.c(om|hm) .bat .pif .s(ys|cr) .ppt .pps .lnk não serão recebidos
nem enviados como anexos em e-mails.

Nome do arquivo:

   anexo-winmail.dat-3bdd4bfe.8R

O nome original do anexo removido é:

   winmail.dat

Caso você precise deste arquivo, favor enviar um e-mail para
[EMAIL PROTECTED] solicitando o arquivo pelo nome:
   anexo-winmail.dat-3bdd4bfe.8R


Informática-PRT15ªRegião

***


_
Get your FREE download of MSN Explorer at http://explorer.msn.com/intl.asp




Re: Terceira fase da OBM

2001-10-22 Por tôpico Marcelo Souza

Gotaria de parabenizar a prova deste ano, principalmente as questoes de 
geometria que estavam belissimas.
[]`s M


From: Nicolau C. Saldanha [EMAIL PROTECTED]
Reply-To: [EMAIL PROTECTED]
To: [EMAIL PROTECTED]
Subject: Re: Terceira fase da OBM
Date: Mon, 22 Oct 2001 09:32:13 -0200

On Sat, Oct 20, 2001 at 07:10:48PM -0200, [EMAIL PROTECTED] wrote:
  Só uma curiosidade : até q idade pode se participar do torneio das 
cidades ?
  Um universitário pode participar ??
  Falow

Não tenho certeza quanto a o que exatamente o regulamento diz quanto
a idades mas universitários não participam. Aqui no Rio convidamos
os classificados para a 3a fase, níveis 1, 2 e 3 a participarem. []s, N.


_
Get your FREE download of MSN Explorer at http://explorer.msn.com/intl.asp




Re: Desigualdades

2001-10-17 Por tôpico Marcelo Souza

coeh
Sera que o fato do grau deve alterar alguma coisa?
De acordo com o livro em que estou estudando, sempre quando acontece uma 
homogenea de grau 1, a suposicao e a+b+c=1 (ou com mais variaveis)
Quando acontece uma de grau maior, o livro sempre usa a=1, b=1+x, c=1+y 
Sei que pode nao ter nada a ver, maseu ate entendi a definicao de 
homogenea pelo livro, a minha duvida era exatamente com relacao a 
substituicao.
se alguem puder esclarecer legal,
valeu!
[]`s M


From: Rodrigo Villard Milet [EMAIL PROTECTED]
Reply-To: [EMAIL PROTECTED]
To: [EMAIL PROTECTED]
Subject: Re: Desigualdades
Date: Wed, 17 Oct 2001 00:03:10 -0300

Uma desigualdade é dita simétrica se ao trocar de ordem as variáveis a 
desigualdade não se altera.
Ex.: a^2 + b^2 + c^2 = ab+ac+bc.
OBS: É interessante termos uma desigualdade simétrica nas variáveis, pois 
podemos supor sem perda de generalidade que elas estão numa certa ordem. No 
exemplo que eu dei, vc pode supor a =b =c ( é claro que há 1001 maneiras 
de provar essa desigualdade sem isso ).

Agora, vamos olhar para desigualdades de outra maneira. Deixe todas as 
variáveis de um lado da inequação. Desse lado tem-se uma função de várias 
variáveis.
Ex.: Em a^2 + b^2 + c^2 = ab+ac+bc, faça F(a,b,c) = a^2 + b^2 + c^2 - 
ab-ac-bc. Vc quer provar que F(a,b,c)=0, para quaisquer a,b,c.
Uma função é dita homogênea de grau n, quando f(ta,tb,tc)=t^n * f(a,b,c).
A desigualdade acima é então homogênea  de grau 2.

Eu acho que o grau não importa muito. O que interessa é se ela é homogênea 
ou não.
Por exemplo, na desigualdade acima, note que F(ta,tb,tc)=0 se e somente se 
F(a,b,c)=0. Então podemos fazer algumas normalizações ( fizar a soma das 
variáveis, fixar uma das variáveis, etc...).
No exemplo dado, faça a=1, b=1+x, c=1+y. Ficamos com 
F(1,1+x,1+y)=x^2+y^2-xy=(x-y/2)^2 + (3y^2)/4 =0.

Outro exemplo bastante significativo é o problema 2 desta última IMO. Era 
uma desigualdade homogênea ( de grau 0, o que não importa ). Daí, era legal 
fazer a+b+c=1, o que nos possibilitava usar a desigualdade de Jensen... e 
assim vai. A moral da história é : fique feliz se a desigualdade for 
simétrica ou homogênea, pois você ou pode matar o problema direto, ou pode 
cair num problema mais fácil. :)

Espero não ter errado alguma definição,
Abraços,
Villard
 -Mensagem original-
 De: Marcelo Souza [EMAIL PROTECTED]
 Para: [EMAIL PROTECTED] [EMAIL PROTECTED]
 Data: Terça-feira, 16 de Outubro de 2001 19:40
 Assunto: Desigualdades


 ol[a pessoal,

 Quando que uma desigualdade e simetrica (acho que diz simetrica em 
relacao as variaveis)?

 Quando uma desigualdade e homogenea de grau n?

 abracos




 Get your FREE download of MSN Explorer at http://explorer.msn.com



_
Get your FREE download of MSN Explorer at http://explorer.msn.com/intl.asp




Re: Desigualdades (correcao)

2001-10-17 Por tôpico Marcelo Souza

cara, outra coisa que nao tinha reparadoto mandando agora...
Eu acho que naum existe homogenea de grau zerode acordo com o livro
f(a,b,c)= f(ta,tb,tc)=tf(a,b,c), com t dif de zero, o minimo e grau 1.
valeu!
corrijam se eu estiver errado
abracos
M.



From: Rodrigo Villard Milet [EMAIL PROTECTED]
Reply-To: [EMAIL PROTECTED]
To: [EMAIL PROTECTED]
Subject: Re: Desigualdades
Date: Wed, 17 Oct 2001 00:03:10 -0300

Uma desigualdade é dita simétrica se ao trocar de ordem as variáveis a 
desigualdade não se altera.
Ex.: a^2 + b^2 + c^2 = ab+ac+bc.
OBS: É interessante termos uma desigualdade simétrica nas variáveis, pois 
podemos supor sem perda de generalidade que elas estão numa certa ordem. No 
exemplo que eu dei, vc pode supor a =b =c ( é claro que há 1001 maneiras 
de provar essa desigualdade sem isso ).

Agora, vamos olhar para desigualdades de outra maneira. Deixe todas as 
variáveis de um lado da inequação. Desse lado tem-se uma função de várias 
variáveis.
Ex.: Em a^2 + b^2 + c^2 = ab+ac+bc, faça F(a,b,c) = a^2 + b^2 + c^2 - 
ab-ac-bc. Vc quer provar que F(a,b,c)=0, para quaisquer a,b,c.
Uma função é dita homogênea de grau n, quando f(ta,tb,tc)=t^n * f(a,b,c).
A desigualdade acima é então homogênea  de grau 2.

Eu acho que o grau não importa muito. O que interessa é se ela é homogênea 
ou não.
Por exemplo, na desigualdade acima, note que F(ta,tb,tc)=0 se e somente se 
F(a,b,c)=0. Então podemos fazer algumas normalizações ( fizar a soma das 
variáveis, fixar uma das variáveis, etc...).
No exemplo dado, faça a=1, b=1+x, c=1+y. Ficamos com 
F(1,1+x,1+y)=x^2+y^2-xy=(x-y/2)^2 + (3y^2)/4 =0.

Outro exemplo bastante significativo é o problema 2 desta última IMO. Era 
uma desigualdade homogênea ( de grau 0, o que não importa ). Daí, era legal 
fazer a+b+c=1, o que nos possibilitava usar a desigualdade de Jensen... e 
assim vai. A moral da história é : fique feliz se a desigualdade for 
simétrica ou homogênea, pois você ou pode matar o problema direto, ou pode 
cair num problema mais fácil. :)

Espero não ter errado alguma definição,
Abraços,
Villard
 -Mensagem original-
 De: Marcelo Souza [EMAIL PROTECTED]
 Para: [EMAIL PROTECTED] [EMAIL PROTECTED]
 Data: Terça-feira, 16 de Outubro de 2001 19:40
 Assunto: Desigualdades


 ol[a pessoal,

 Quando que uma desigualdade e simetrica (acho que diz simetrica em 
relacao as variaveis)?

 Quando uma desigualdade e homogenea de grau n?

 abracos




 Get your FREE download of MSN Explorer at http://explorer.msn.com



_
Get your FREE download of MSN Explorer at http://explorer.msn.com/intl.asp




Re: Desigualdades (correcao)(desculpas)

2001-10-17 Por tôpico Marcelo Souza


ih tahdesculpa mesmo vc se referia ao nestava desligadao 
mesmo...foi mal cara...e o livro do arthur engel que eu olhei 
mesmo...hehehehe
Tava desatento, foi mal.
[]`s, M

From: Rodrigo Villard Milet [EMAIL PROTECTED]
Reply-To: [EMAIL PROTECTED]
To: [EMAIL PROTECTED]
Subject: Re: Desigualdades (correcao)
Date: Wed, 17 Oct 2001 15:59:20 -0300

grau zero, pois fica t^0. a da IMO era assim :

a/sqrt(a^2+8bc) + b/sqrt(b^2+8ac) + c/sqrt(c^2+8ba) =1

Quando vc calcula F(ta,tb,tc) dá exatamente F(a,b,c), ou seja o grau é 
zero.
Você pode olhar no livro Problem-Solving Strategies, Arthur Engel. Ele dá
mais um exemplo :
Prove que a,b,c0  a/(b+c) + b/(a+c) + c/(a+b) = 3/2.
Ele mesmo diz que F(a,b,c) é homogênea de grau 0 e nesse caso ele faz a
normalização a+b+c=1, que é bem natural pois aparecem as somas parciais.
Então use a desigualdade das médias aritmética e harmônica com os números
a+b, a+c, b+c. Daí (a+b+a+c+b+c)(1/(a+b) + 1/(a+c) + 1/(b+c)) =9... ou 
seja
:  1/(a+b) + 1/(a+c) + 1/(b+c) = 9/2 logo (a+b+c)/(a+b) + (a+b+c)/(a+c) +
(a+b+c)/(b+c) =9/2 ...
  a/(b+c) + b/(a+c) + c/(a+b) = 3/2 (CQD)
  Talvez quando vc perceber que a desigualdade é homogênea vc nem tenha que
citar o grau... a substituição que faz é o mais importante...
  Mas vc percebeu pq pode normalizar ? é que (a,b,c) satisfaz a
desigualdade se, e somente se, (ta,tb,tc) tb satisfaz (isso é bastante 
claro
qd o grau é zero). Daí, se vc supõe por exemplo a+b+c = 1 e prova a
desigualdade, vc está provando q vale a desigualdade para qualquer soma
a+b+c. Mesma coisa para o artifício a=1, b=1+x, c=1+y... basta olhar para o
que acontece com o a.
Me corrijam se estiver errado.
Abraços, Villard

PS.: Em qual livro vc está estudando isso ?



-Mensagem original-
De: Marcelo Souza [EMAIL PROTECTED]
Para: [EMAIL PROTECTED] [EMAIL PROTECTED]
Data: Quarta-feira, 17 de Outubro de 2001 14:09
Assunto: Re: Desigualdades (correcao)


 cara, outra coisa que nao tinha reparadoto mandando agora...
 Eu acho que naum existe homogenea de grau zerode acordo com o livro
 f(a,b,c)= f(ta,tb,tc)=tf(a,b,c), com t dif de zero, o minimo e grau 1.
 valeu!
 corrijam se eu estiver errado
 abracos
 M.
 
 
 
 From: Rodrigo Villard Milet [EMAIL PROTECTED]
 Reply-To: [EMAIL PROTECTED]
 To: [EMAIL PROTECTED]
 Subject: Re: Desigualdades
 Date: Wed, 17 Oct 2001 00:03:10 -0300
 
 Uma desigualdade é dita simétrica se ao trocar de ordem as variáveis a
 desigualdade não se altera.
 Ex.: a^2 + b^2 + c^2 = ab+ac+bc.
 OBS: É interessante termos uma desigualdade simétrica nas variáveis, 
pois
 podemos supor sem perda de generalidade que elas estão numa certa ordem.
No
 exemplo que eu dei, vc pode supor a =b =c ( é claro que há 1001 
maneiras
 de provar essa desigualdade sem isso ).
 
 Agora, vamos olhar para desigualdades de outra maneira. Deixe todas as
 variáveis de um lado da inequação. Desse lado tem-se uma função de 
várias
 variáveis.
 Ex.: Em a^2 + b^2 + c^2 = ab+ac+bc, faça F(a,b,c) = a^2 + b^2 + c^2 -
 ab-ac-bc. Vc quer provar que F(a,b,c)=0, para quaisquer a,b,c.
 Uma função é dita homogênea de grau n, quando f(ta,tb,tc)=t^n * 
f(a,b,c).
 A desigualdade acima é então homogênea  de grau 2.
 
 Eu acho que o grau não importa muito. O que interessa é se ela é 
homogênea
 ou não.
 Por exemplo, na desigualdade acima, note que F(ta,tb,tc)=0 se e somente
se
 F(a,b,c)=0. Então podemos fazer algumas normalizações ( fizar a soma 
das
 variáveis, fixar uma das variáveis, etc...).
 No exemplo dado, faça a=1, b=1+x, c=1+y. Ficamos com
 F(1,1+x,1+y)=x^2+y^2-xy=(x-y/2)^2 + (3y^2)/4 =0.
 
 Outro exemplo bastante significativo é o problema 2 desta última IMO. 
Era
 uma desigualdade homogênea ( de grau 0, o que não importa ). Daí, era
legal
 fazer a+b+c=1, o que nos possibilitava usar a desigualdade de Jensen... 
e
 assim vai. A moral da história é : fique feliz se a desigualdade for
 simétrica ou homogênea, pois você ou pode matar o problema direto, ou 
pode
 cair num problema mais fácil. :)
 
 Espero não ter errado alguma definição,
 Abraços,
 Villard
  -Mensagem original-
  De: Marcelo Souza [EMAIL PROTECTED]
  Para: [EMAIL PROTECTED] [EMAIL PROTECTED]
  Data: Terça-feira, 16 de Outubro de 2001 19:40
  Assunto: Desigualdades
 
 
  ol[a pessoal,
 
  Quando que uma desigualdade e simetrica (acho que diz simetrica em
 relacao as variaveis)?
 
  Quando uma desigualdade e homogenea de grau n?
 
  abracos
 
 
 
 --
--
  Get your FREE download of MSN Explorer at http://explorer.msn.com
 
 
 
 _
 Get your FREE download of MSN Explorer at 
http://explorer.msn.com/intl.asp
 



_
Get your FREE download of MSN Explorer at http://explorer.msn.com/intl.asp




Desigualdades

2001-10-16 Por tôpico Marcelo Souza

ol[a pessoal,
Quando que uma desigualdade e simetrica (acho que diz simetrica em relacao as variaveis)?
Quando uma desigualdade e homogenea de grau n?
abracosGet your FREE download of MSN Explorer at http://explorer.msn.com


Re: Onde compo o Saraeva?

2001-10-15 Por tôpico Marcelo Souza

coeh igorvc so esqueceu de mencionar que no seu caso o professor e seu paiai fica bem mais facila=)
abra;os, M.

From: "Igor Castro" <[EMAIL PROTECTED]>
Reply-To: [EMAIL PROTECTED] 
To: <[EMAIL PROTECTED]>
Subject: Re: Onde compo o Saraeva? 
Date: Sun, 14 Oct 2001 21:32:15 -0300 
 
Realmente vai ser muito dificil você achar ele para vender, mesmo em sebos, acho que a melhor forma é você tentar conseguir com algum professor, assim eu consegui dois originais... em ultimo caso você pode xerocar de alguém, porque realmente vale a pena... 
[]'s 
 - Original Message - 
 From: Gustavo Nunes Martins 
 To: [EMAIL PROTECTED] 
 Sent: Saturday, October 13, 2001 5:02 PM 
 Subject: Onde compo o Saraeva? 
 
 
 
 
 Dizem que ha um livro de fisica chamado Saraeva e que ele e tem questoes 
 muito interessantes. O titulo dele e Saraeva mesmo ou esse e o nome do 
 autor? Eu procurei esse livro na pagina de uma editora que parece ter 
 pego os livros da MIR (www.urss.ru), mas nao achei nada (nao sei de qual 
 editora esse livro e). 
 
 Onde eu acho esse livro? Ele e tao bom assim? 
 
 
 Obrigado, 
 Gustavo 
 
 
Get your FREE download of MSN Explorer at http://explorer.msn.com


Re: Onde compo o Saraeva?

2001-10-14 Por tôpico Marcelo Souza


Fala cara..
O livro do saraeva  simplesmente o melhor livro de exerccios de fsica... Acontece que ele  da editora MIR e no  mais publicado... muito difcil ach-lo e s h em sebos. Do meu curso, que tem mais de 500 alunos, apenas 1 deles tinha o livroInclusive combinamos de procurar em sebos, mas no encontramos nadaa melhor forma de adquiri-lo seria xerocando de algum, porque achar.tem que ser sortudo.
[]'s, M.

From: Gustavo Nunes Martins<[EMAIL PROTECTED]> 
Reply-To: [EMAIL PROTECTED] 
To: [EMAIL PROTECTED]<[EMAIL PROTECTED]> 
Subject: Onde compo o Saraeva? 
Date: Sat, 13 Oct 2001 17:02:29 -0300 
 
Dizem que ha um livro de fisica chamado Saraeva e que ele e tem questoes 
muito interessantes. O titulo dele e Saraeva mesmo ou esse e o nome do 
autor? Eu procurei esse livro na pagina de uma editora que parece ter 
pego os livros da MIR (www.urss.ru), mas nao achei nada (nao sei de qual 
editora esse livro e). 
 
Onde eu acho esse livro? Ele e tao bom assim? 
 
 
Obrigado, 
Gustavo 
 
Get your FREE download of MSN Explorer at http://explorer.msn.com


Re: Problema sobre primos

2001-10-08 Por tôpico Marcelo Souza

Legal, não sabia que já existiam fórmulas que geravam primos e somente 
primos...
Minhas desculpas ao Eric.
=)
[]'s, M.


From: Paulo Jose Rodrigues [EMAIL PROTECTED]
Reply-To: [EMAIL PROTECTED]
To: [EMAIL PROTECTED]
Subject: Re: Problema sobre primos
Date: Mon, 8 Oct 2001 16:15:15 -0300

  Se vc resolveu esse problema, vc deveria dar conferencias no mundo
  inteirovc seria maior que Gauss ou EinsteinTal formula não
existe!!!
  Um abraço e  não se engane...essa lista não tem ingênuos...
 Ruy

Ruy,
  existem infinitas fórmulas que geram somente números primos. O que 
acontece
é que a grande maioria dessas fórmulas são inúteis do ponto de vista
prático.

Por exemplo, não é difícil provar com a ajuda do Teorema de Wilson, que a
função

f(x, y)=(y-1)/2[|B^2-1|-(B^2-1)]+2,

onde B=x(y+1)-(y!+1), x e y são números naturais, gera somente números
primos, gera todos os primos e gera todos os primos ímpares exatamente uma
vez.





_
Get your FREE download of MSN Explorer at http://explorer.msn.com/intl.asp




Re: análise combinatória

2001-10-01 Por tôpico Marcelo Souza


Eu tenho muita coisa aki, se vc quser...
Se vc tiver icq...uin 57193686
Caso naum, mande um email
[]'s, M

From: pichurin pichurin<[EMAIL PROTECTED]> 
Reply-To: [EMAIL PROTECTED] 
To: [EMAIL PROTECTED] 
Subject: anlise combinatria 
Date: Sun, 30 Sep 2001 21:12:06 -0300 (ART) 
 
pessoal, onde poderia encontrar problemas de anlise 
combinatria com respostas na net? 
vcs poderiam me mandar alguns? 
 
___ 
Yahoo! GeoCities 
Tenha seu lugar na Web. Construa hoje mesmo sua home page no Yahoo! GeoCities.  fcil e grtis! 
http://br.geocities.yahoo.com/ 
Get your FREE download of MSN Explorer at http://explorer.msn.com


Re: Probleminhas

2001-09-20 Por tôpico Marcelo Souza

é como a gente escreve na lista somatório. (a letra grega sigma)
ao inves do digma colocamos SUM


From: [EMAIL PROTECTED]
Reply-To: [EMAIL PROTECTED]
To: [EMAIL PROTECTED]
Subject: Re: Probleminhas
Date: Thu, 20 Sep 2001 20:55:25 -0300

Desculpe-me, mas não entendi o que é SUM (1/j).




Em 19 Sep 2001, [EMAIL PROTECTED] escreveu:

 Alguem pode mandar a soluçao dos problemas?
 
 1.Mostre que
 n
 SUM (1/j) não é um número inteiro.
 j=1
 
 2. Mostre que todo o primo da forma 3k+1 tb é da forma 6k+1.
 valeu
 []'s, M.
 
 _
 Get your FREE download of MSN Explorer at 
http://explorer.msn.com/intl.asp
 
 --

_
Oi! Você quer um iG-mail gratuito?
Então clique aqui: http://registro.ig.com.br/



_
Get your FREE download of MSN Explorer at http://explorer.msn.com/intl.asp




Geo Analitica.

2001-09-20 Por tôpico Marcelo Souza

Galera, alguem poderia explicar pra mim como se faz esse problema ( bem simples, mas no livro a teoria esta um pouco jogada)
Given two points P and Q, a units apart, and the functionf(M)=(d_1)^2-(d_2)^2, where d_1=MP and d_2=MQ. FRind the expressionfor this function when the point P is chosen as the origin, and the axis Ox is directedalong the segment PQ.
valeu
abrao
Get your FREE download of MSN Explorer at http://explorer.msn.com


Probleminhas

2001-09-18 Por tôpico Marcelo Souza

Alguem pode mandar a soluçao dos problemas?

1.Mostre que
n
SUM (1/j) não é um número inteiro.
j=1

2. Mostre que todo o primo da forma 3k+1 tb é da forma 6k+1.
valeu
[]'s, M.

_
Get your FREE download of MSN Explorer at http://explorer.msn.com/intl.asp




Re: para Nicolau

2001-09-14 Por tôpico Marcelo Souza

Nicolau, pra quem fez prova da obm segunda fase na PUC, o resultado sai qdo? 
Meu amigo do CM já obteve respostas.
[]'s, M.


From: Nicolau C. Saldanha [EMAIL PROTECTED]
Reply-To: [EMAIL PROTECTED]
To: [EMAIL PROTECTED]
Subject: Re: Questao 5 do nivel 2=questao 4 do nivel 3
Date: Thu, 13 Sep 2001 14:04:02 -0300

On Thu, Sep 13, 2001 at 01:33:58AM +, Fernanda Medeiros wrote:
 
 Olá,
  Desculpa mas não concordo com a posição tomada pela comissão...Acho que 
ela
  vai mais prejudicar do que ajudar, pois creio que a grande maioria  das
  pessoas identificou mais ou menos o erro e viu que seria perda de tempo
  tentar resolvê-la, de modo que ela foi deixada de ladoDe fato, a
  decisão vai prejudicar não só a mim como a grande maioria das pessoas 
que
  pensou deste mesmo jeito(e com certeza foram muitas pessoas!)
  Fê
 

Você tem todo o direito de discordar e expressar a sua opinião.
Mas já que você o faz eu repito o que o Fred já falou:
não se deve deixar de lado uma questão de olimpíada por
só se saber fazer um ou alguns casos; se você só souber fazer
alguns casos você deve escrever os casos que sabe e deixar claro
que sabe que está fazendo apenas alguns casos e que não teve tempo
para fazer os outros. Muitos alunos nesta questão fizeram exatamente
isto e ganharam 10 pontos: eles claramente merecem receber mais pontos
do que quem não escreveu nada. É claro que eles merecem ganhar *menos*
do que os alunos que corretamente estudaram todos os casos: idealmente,
quem fez o caso principal (ou seja, fez o que está no gabarito oficial
que foi distribuído junto com a prova) deveria receber uma pontuação
intermediária, talvez 7 pontos, e só deveria ganhar 10 pontos quem
estudou todos os casos --- seria, entretanto, inviável transmitir esta
informação a tempo para as escolas e também seria inviável recorrigir
todas as provas. Note, aliás, que existem vários casos de alunos que
estudaram corretamente todos os casos. Se eu bem entendi, a sua opinião
é a de que a questão deveria ser anulada: esta não foi a opinião de nenhum
membro da comissão.

De qualquer forma, boa sorte para você e para todos os participantes!

[]s, N.


_
Get your FREE download of MSN Explorer at http://explorer.msn.com/intl.asp




Re: Questao 5 do nivel 2=questao 4 do nivel 3

2001-09-13 Por tôpico Marcelo Souza

Oi, André, desculpe retornar o email, mas acho que a segunda pergunta foi 
bem geral. Acho que ano passado o corte foi 58 (acho). Esse ano, um 
professor amigo meu que trabalha na UFRJ diz que acha que deve estar em 
torno de 52 (sem nenhuma certeza).
Valeu
abraços, M.
PS. 58 bla bla somado com a primeira fase, claro


From: Andre S [EMAIL PROTECTED]
Reply-To: [EMAIL PROTECTED]
To: [EMAIL PROTECTED]
Subject: Re: Questao 5 do nivel 2=questao 4 do nivel 3
Date: Wed, 12 Sep 2001 22:05:56 -0300 (ART)

Nicolau,

Realmente, devido às complicações envolvidas, essa
decisão passa a ser muito melhor que as outras... Mas,
(lá vem o mas...) pensando um pouco, você não acha que
o caso principal é uma particularidade, que incita
perda da generalidade da questão se não forem
trabalhadas as outras hipóteses de combinação? Nem é
meu caso, só pura curiosidade, mas eu tive a impressão
que as outras soluções, dado o enunciado, seriam mais
importantes que o caso particular, valendo mais
pontos...

(Não há pressa, já que é apenas curiosidade)

André
PS: Qual foi a média de corte ano passado para o 3º nível?

___
Yahoo! GeoCities
Tenha seu lugar na Web. Construa hoje mesmo sua home page no Yahoo! 
GeoCities. É fácil e grátis!
http://br.geocities.yahoo.com/


_
Get your FREE download of MSN Explorer at http://explorer.msn.com/intl.asp




Re: ajuda

2001-09-09 Por tôpico Marcelo Souza

Concordo com o Bruno! Paciência, as pessoas da lista se esforçam para fazer 
os problemas, mas nós não estamos fazendo somente isso das nossas vidas. 
Grande parte aqui leciona, temos que esperar uma brechinha para que possamos 
faze-los...por isso compreenda. Não fique enviando o mesmo problema centenas 
de vezes...isso atrapalha inclusive a diversidade da lista e não adianta em 
nada.
valeu
abraços, Marcelo!


From: Bruno Fernandes Cerqueira Leite [EMAIL PROTECTED]
Reply-To: [EMAIL PROTECTED]
To: [EMAIL PROTECTED]
Subject: Re: ajuda
Date: Sun, 09 Sep 2001 12:43:59 -0300

de novo este problema??

Bruno


At 07:50 09/09/01 EDT, [EMAIL PROTECTED] wrote:
 Ao estudar as ciências, os exercícios são mais úteis do que as regras
  Assim escreveu Isaac Newton em sua aritmética Universal e, de fato,
 acompanhava as indicações teóricas com uma série de exemplos. No meio
deles,
 acha-se o problema das vacas que pastam em uma campina, exemplo esse que
deu
 origem a um tipo específico de problemas semelhantes a ele.
 
  O capim cresce no pasto todo com igual velocidade e espessura. 
Sabe-se
 que 70 vacas o comeriam em 24 dias; 30 vacas em 60 dias. Quantas vacas
 comeriam todo o capim em 96 dias?
  Duas pessoas, tentando resolver este problema, esforçavam-se por
 descobrir sua resposta.
  - Que resultado esquisito! - disse um deles.
  - Se em 24 dias, 70 vacas comem o capim todo, quantas comeriam em 96
 dias? Está visto que é 1/4 de 70, isto é, 17,5 vacas... Este é o primeiro
 absurdo! o segundo, mais esquisito ainda, é que se 30 vacas comem o capim
em
 60 dias, em 96 dias o capim será comido por 18,75 vacas. E ainda por 
cima,
se
 70 vacas comem em 24 dias, 30 gastarão nisso 56 dias e não 60, como 
afirma
o
 problema.
  - Mas você está levando em conta que o capim cresce sem parar? -
 perguntou o outro.
  A observação procedia. A grama cresce sem cessar, fato esse que não
 pode ser esquecido, pois que, se o fizéssemos, não só o problema ficaria
sem
 solução, mas as suas próprias condições pareceriam contraditórias.
  Podemos então concluir que 21 vacas comeriam o capim em quantos 
dias?
 
 Aguardo comentários. Grato!!



_
Get your FREE download of MSN Explorer at http://explorer.msn.com/intl.asp




Re: Lista de discussão sobre fisica

2001-09-08 Por tôpico Marcelo Souza


Puxa, eu naum sei se estaria certo, mas...Esta listaaqui acho que o pessoal 
libera a fisica legal...já vi conversando sobre fisicaagora 
especificamente de fisica naum sei
[]'s, M.

From: romenro[EMAIL PROTECTED]
Reply-To: [EMAIL PROTECTED]
To: [EMAIL PROTECTED]
Subject: Lista de discussão sobre fisica
Date: Sat,  8 Sep 2001 17:12:57 -0300

Eu gostaria de saber se alguém conhece uma lista de
discussão como essa da OBM só que de física?

Desculpe fugir do assunto que é o objetivo dessa lista,
mas acredito que voces compreendem.

Desde já agradeço,

Rodrigo.




__
AcessoBOL, só R$9,90! Menos que a metade da América!
Assine já! http://www.bol.com.br/acessobol/




_
Get your FREE download of MSN Explorer at http://explorer.msn.com/intl.asp




Re: Questão 4 OBM-2001 Nivel 3

2001-09-07 Por tôpico Marcelo Souza

Fala Lucas! Blz?
Creio eu (corrijam-me se eu estiver errado) que não ´há como perder pontos 
por naum demonstrar as outras hipoteses NESTE caso. Pois até mesmo o 
gabarito naum demonstrou nenhuma hipotese. Também pensei nas outras 
hipoteses que poderiam existir, mas eu escrevi exatamente como está no 
enunciado, provei aquele ac=bdseja lah como for, como achei que ia haver 
outra hipotese separei em letra (a), mas como naum teve (b) ficou por isso 
mesmo...Como naum havia outra hipotese no gabarito quero meu ponto integral 
=)
abraços
Marcelouin 57193686


From: Lucas Povarczuk Mocelim [EMAIL PROTECTED]
Reply-To: [EMAIL PROTECTED]
To: [EMAIL PROTECTED]
Subject: Re: Questão 4 OBM-2001 Nivel 3
Date: Fri, 7 Sep 2001 18:07:20 -0300



Olá amigos da lista!

Realmente, a questão 4 da prova do nível 3 (que também caiu no nível 2) não
dizia que valia apenas o caso (10a+b)(10c+d) = (10b+a)(10d+c) para um
conjunto A = {a,b,c,d} ser intercambiável (para simplificar, vou assumir
como sendo esse o caso AB x CD = BA x DC). Quando escrevi a solução na hora
da prova, mostrando as 5 soluções, olhei pro papel e pensei feito, mais
uma, beleza!. Mas aí veio aquela idéia que vocês também tiveram (diga-se 
de
passagem, bastante pertinente): xiii... mas há outras hipóteses que eu não
mostrei! Vou perder muitos pontos! Ah não, vou ter que escrever cada
hipótese e verificar que são absurdas...

Então, mostrei que, sem perda de generalidade, igualando {AB x CD} aos
produtos {BC x AD}, {BC x DA}, {BD x AC} e {BD x CA}, em todos eles há um
absurdo que inviabiliza soluções, havendo apenas soluções para o caso
apresentado no enunciado, AB x CD = BA x DC.

Demorei vários minutos provando esses casos, e creio que muita gente deve
ter desistido da questão por ter justamente imaginado que haveria outros
casos a analisar. Na hora que me dei conta das outras possibilidades, achei
que não haveria tempo pra fazê-las todas.

Não sei se seria correto anular a questão, como acha o colega Jorge 
Peixoto,
até porque alguns estudantes conseguiram verificar todos os casos. A minha
opinião é que, se o problema foi feito dessa forma, na solução da banca
deveria constar uma análise mais completa.

Espero eventuais críticas ou opiniões. No mais, boa sorte a todos!

Um abraço a todos os amigos,

Lucas

___

 A questão é ambígua sem dúvida.
 
 No dia da prova, eu tive que resolver as 11 possibilidades de equações 
para
 verificar se havia outras soluções além das citadas pelo gabarito 
oficial.
 Mas, o mais estranho, é que apenas a equação (10x + y)(10a + b) = (10y +
 x)(10b + a) tem solução. Ou seja, apenas o caso particular citado pela
 correção
 serve, mas, na prova, tive que me preocupar em justificar tudo que
escrevia,
 e, claro, porque apenas aquelas soluções ocorrem. É claro que isso me fez
 perder muito tempo de prova.
 -- Mensagem original --
 
 Questão 4 OBM-2001 Nivel 3 está ambígua??
 
 Concordo plenamente... Eu e mais três alunos do meu colégio leram e
fizeram
 a questão como se pudesse escolher , por exemplo: dentre os 4 números x,
 y,
 z e w uma das possibilidades da quadrúpla ser intercambiável era
 (10x+y)(10z+w)=(10x+w)(10y+z) o que tornaria a solução do problema
original
 apenas um caso particular... Tornando o problema bem mais difícil!! Além
 disso ao falar com outros alunos de outros colégios vi que ocorreram 
casos
 iguais a esses... E ocorreu até em outros lugares no caso do Henrique
 Noguchi... Creio que o enunciado do problema 4 do nível 3 esteja 
ambíguo!!
 E
 além disso nenhum desses alunos que o interpretaram de maneira diferente
 conseguiu terminar a solução desse caso generalizado em tempo de prova
 (pelo menos aqui em Fortaleza)!! Acho que antes de dar algum parecer 
sobre
 esse caso deve-se discutir muito para que nenhum aluno seja 
prejudicado!!
 
 Obrigado pela atenção!!
 EINSTEIN
 
 -Mensagem original-
 De: [EMAIL PROTECTED] [mailto:[EMAIL PROTECTED]]Em
 nome de Nicolau C. Saldanha
 Enviada em: quarta-feira, 5 de setembro de 2001 09:01
 Para: [EMAIL PROTECTED]
 Assunto: Re: Questão 5 OBM-2001 Nivel 2
 
 
 On Tue, Sep 04, 2001 at 09:16:13PM -0300, Vanda Noguchi wrote:
  Na questão 5 da última OBM (2001), a solução do gabarito da OBM assume
 que
  os números são formados pelos mesmos digitos trocando de posição, tal
 como
  (21 e 12) ou (36 e 63) ou seja, (10x + y)(10t + z) = (10y + x)(10z +
 t).
  O exemplo dado na questão está desta forma, mas nada no enunciado leva
 a
  concluir isto. A equação acima não abrange os números (10x+t), 
(10x+z),
  (10z+y), etc..Portanto, a solução do gabarito é uma particularidade do
  enunciado. Alguém consegue explicar se minha conclusão é correta?
 
 Esta situação está sendo discutida pela comissão de olimpíadas
 e teremos uma posição oficial em breve, provavelmente hoje ou amanhã. 
[]s,
 N.
 
  A questão é a seguinte:
  Dizemos que um conjunto A formado por 4 algarismos distintos e não
nulos
 é
  intercambiável se podemos formar dois 

Re: Ambiguidade

2001-09-06 Por tôpico Marcelo Souza


Engraçado, sinceramente, eu consegui provar que os algarismos teriam que dar 
probudtos iguais (dois a dois) para que fossem intercambiáveis, porém, eu 
sabia que naum dava pra montar outro caso, pois dava alguma loucura. Não 
escrevi na minha prova, pois achei desnecessário. Também fiquei encucado com 
a comissão...até mandei um email anterior em relação a isso...mas acho que 
naum vão implicar com isso porque na solução do gabarito não há nenhuma 
outra forma...
O meu problema foi na cinco, a questão tava legal até eu chegar na parte 
daquela observação final em que combinações daqueles tipos deveriam ser 
consideradas identicas...Acontece que como as figuras de cabeça pra baixo 
eram as mesmas, interpretei que todas que tivessem tal propriedade deveriam 
ser consideradas iguais...Deu uma coisa tão absurda!!
[]'s, M.

From: Odelir Maria Casanova dos Santos [EMAIL PROTECTED]
Reply-To: [EMAIL PROTECTED]
To: [EMAIL PROTECTED]
Subject: Ambiguidade
Date: Thu, 6 Sep 2001 14:54:17 -0300

 Eu também concordo que a questão 4 do nível três estava ambígua, 
quando tentei fazer-la durante a prova sabia que 
havia outras possibilidades como ( 10a + y )( 10b + x ) = ( 10a + x  )( 10y 
+ b ), só que neste momento percebi que daria uma solução MUITO 
mais complexa do que deveria e como não me havia sobrado muito tempo, logo
a deixei sem resolução ( na verdade não me sobrou muito tempo para outras 
mesmo... ).
 Na meu colégio creio que ninguém tenha tido idéia para resolve-la, 
aliás no meu colégio acho que ninguém ( inclusive eu ! )
teve idéia para resolver as três últimas questões.
 Agora é só esperar a posição da comissão organizadora para termos a 
pontuação final da prova.

Esperando...

Marcus Dimitri


_
Get your FREE download of MSN Explorer at http://explorer.msn.com/intl.asp




Re: Ajuda para achar um livro

2001-09-06 Por tôpico Marcelo Souza

Na livraria da Rua do Rosário, no rio, centro da cidade. Descendo na estação 
de Uruguaiana no metro fica pertinho pra chegar. É só perguntar onde que é a 
rua do Rosário por ali e pronto.
[]'s, M.


From: Luis Lopes [EMAIL PROTECTED]
Reply-To: [EMAIL PROTECTED]
To: [EMAIL PROTECTED]
CC: [EMAIL PROTECTED]
Subject: Ajuda para achar um livro
Date: Thu, 6 Sep 2001 18:06:24 -0300

Sauda,c~oes,

Será que alguém teria alguma coisa
a dizer?

Um abraço,
Luís

  -- Forwarded Message
  From: rodferro[EMAIL PROTECTED]
  Date: Wed,  5 Sep 2001 00:17:24 -0300
  To: [EMAIL PROTECTED]
  Subject: duvida-
 
  Caro professor,
  estava lendo o livro Manual de progressoes,(que eh muito
  bom ,parabens),quando eu deparei na bibliografia o
  seguinte livro:
  Shlarsky,D.O.,Chentzov,N.N,and Yaglom,I.M,The USSR
  Olympiad Problem Book,W.H Freeman,1962.
  Estou procurando por esse livro faz tempo,ja procurei em
  sebo ,em varias bibliotecas,enfim muitos lugares.O
  senhor saberia me informar onde posso compra-lo,ou
  consulta-lo.(Pode ser tanto no rio ou em sp).
  Muito Obrigado.
  Rodrigo.
  (Parabens mais uma vez pelo livro).
  -- End of Forwarded Message
 



_
Get your FREE download of MSN Explorer at http://explorer.msn.com/intl.asp




Correção da OBM!

2001-09-05 Por tôpico Marcelo Souza

Oi, nessa segunda fase eu consegui acertar 5 questes. Algo me preocupa: um amigo meu me disse que na correo eles tiram ponto se vc no fez certas observaes. Ele falou que h um roteiro e que cada ponto que vc ganha  seguido pelo roteiro.que que isso quer dizer? Tipo, mesmo se eu acertar a questo eu posso perder ponto porque eu fiz algo que no estava no roteiro?
Obrigado
Abraos
MarceloGet your FREE download of MSN Explorer at http://explorer.msn.com


Boa Sorte

2001-08-31 Por tôpico Marcelo Souza
E aí, pessoal? Beleza?
Estou desejando boa sorte pro pessoal da prova de amanhã...sorte é uma boa coisa...mas sendo um pouquinho diferente, muita paz e tranquilidade pra fazer a prova de amanhã, isso é muito importante...
Um grande abraço pra todos
Marcelo!Get your FREE download of MSN Explorer at http://explorer.msn.com


Re: maio

2001-08-19 Por tôpico Marcelo Souza


O primeiro parece ser bem simples. Monte sua figura. Note que como ABC  isosceles AN e CM so alturas. Fixe no quadriltero APCM, notou que ele  inscritivel? Poemos afirmar que os angulos PCA = AMP = 45 e PMC = PAC = 45, isso mostra que MP  bissetriz do do angulo AMC (triangulo AMC), como AN tambm  bissetriz, concluimos que I  o incentro do triangulo AMC, tah provado
[]'s M.

From: Henrique Lima<[EMAIL PROTECTED]> 
Reply-To: [EMAIL PROTECTED] 
To: [EMAIL PROTECTED] 
Subject: maio 
Date: Mon, 20 Aug 2001 02:25:01 + 
 
 
 
 
estas questes foram da olimpiada de maio de 99, alguem pode 
resolv- 
-las? (pra eu comparar com minhas repostas, pois no tenho 
certeza...) 
 
1.Seja ABC um triangulo equiltero.M eh o pnt mdio de AB e N o pnt 
mdio de 
BC.Seja P o pnt exterior a ABC tal que o triangulo ACP  issceles e 
retangulo em P.PM e AN cortam-se em I.Prove q CI eh a bissetriz do 
angulo 
MC^A. 
2.Um n natural de 3 dgitos eh chamado tricubico se eh igual a soma 
dos 
cubos dos seus dgitos.Encontre todos os pares de numeros 
consecutivos tais 
que ambos sejam tricubicos. 
3. Num paralelogramo ABCD,BD eh a diagonal maior. Ao fazer coincidir 
B com 
D, mediante uma dobra, forma-se um pentgono regular.Calcular as 
medidas 
dos angulos q a diagonal BD forma com cada um dos lados do 
paralelogramo. 
 valeu! 
 
 
_ 
Get your FREE download of MSN Explorer at 
http://explorer.msn.com/intl.asp 
 
Get your FREE download of MSN Explorer at http://explorer.msn.com


Re: maio

2001-08-19 Por tôpico Marcelo Souza


O 3 ... Qdo dobramos para B coincidir com C, dobramos o plano do paralelogramo exatamente na mediatriz de BD, ou seja, tome BD, agora tome seu ponto mdio, agora trace uma reta perpendicular a BD passando por esse ponto mdio. Isto determina dois pontos: um no lado AB e outro no lado CD, chame respectivamente de M e N. Ao dobrar note que os angulos A e C no sofrem altero, logo, eles valem 108 (medida do angulo interno do pentgono). Do ponto B que agora coincide com D trace uma perpendicular a MN no ponto mdio (pois como j vimos este ponto mdio  o ponto mdio de BD antes de dobrarmos), chame-o de H. Ligue BM e BN, repare que os triangulos so isosceles, isso mostra que AMB = 36 e MBH = 18, pronto, um dos angulos  54 que forma com o lado AD, e o outro  18 que forma com o lado CD, valeu!
Desculpa se eu expliquei um pouco confuso, mas  que sem desenhocheio de sono
[]'s M.

From: Henrique Lima<[EMAIL PROTECTED]> 
Reply-To: [EMAIL PROTECTED] 
To: [EMAIL PROTECTED] 
Subject: maio 
Date: Mon, 20 Aug 2001 02:25:01 + 
 
 
 
 
estas questes foram da olimpiada de maio de 99, alguem pode 
resolv- 
-las? (pra eu comparar com minhas repostas, pois no tenho 
certeza...) 
 
1.Seja ABC um triangulo equiltero.M eh o pnt mdio de AB e N o pnt 
mdio de 
BC.Seja P o pnt exterior a ABC tal que o triangulo ACP  issceles e 
retangulo em P.PM e AN cortam-se em I.Prove q CI eh a bissetriz do 
angulo 
MC^A. 
2.Um n natural de 3 dgitos eh chamado tricubico se eh igual a soma 
dos 
cubos dos seus dgitos.Encontre todos os pares de numeros 
consecutivos tais 
que ambos sejam tricubicos. 
3. Num paralelogramo ABCD,BD eh a diagonal maior. Ao fazer coincidir 
B com 
D, mediante uma dobra, forma-se um pentgono regular.Calcular as 
medidas 
dos angulos q a diagonal BD forma com cada um dos lados do 
paralelogramo. 
 valeu! 
 
 
_ 
Get your FREE download of MSN Explorer at 
http://explorer.msn.com/intl.asp 
 
Get your FREE download of MSN Explorer at http://explorer.msn.com


Re: RES: Números...

2001-08-18 Por tôpico Marcelo Souza


Esqueci de mencionar que a,b,c, so inteiros positivos distintos...

From: M. A. A. Cohen<[EMAIL PROTECTED]> 
Reply-To: [EMAIL PROTECTED] 
To:<[EMAIL PROTECTED]> 
Subject: RES: Nmeros... 
Date: Thu, 16 Aug 2001 23:05:16 -0300 
 
a+b=q^2. a,b=1 = q^2 = 2 = q1 = q^2 =.4. 
Analogamente, b+c=l^2 e a+c=r^2 com l^2, r^2 =4. 
Entao, n=(q^2+l^2+r^2)/2 = (4+4+4)/2 = 6. 
Tomando a=b=c=2 vemos que n=6 eh de fato o menor n inteiro positivo de forma 
que  
 
t+ 
Marcio 
 -Mensagem original- 
 De: [EMAIL PROTECTED] [mailto:[EMAIL PROTECTED]]Em nome 
de Marcelo Souza 
 Enviada em: sexta-feira, 17 de agosto de 2001 19:05 
 Para: [EMAIL PROTECTED] 
 Assunto: Nmeros... 
 
 
 Eu gostaria de ver as solues para o problema: 
  Determine o menor inteiro positivo n tal que n = a + b + c, com a,b,c 
inteiros positivos, de forma que a + b, b + c, a + c sejam quadrados 
perfeitos 
 Obrigado 
 []'s M. 
 
 
 
-- 
 Get your FREE download of MSN Explorer at http://explorer.msn.com 
 
Get your FREE download of MSN Explorer at http://explorer.msn.com


Números...

2001-08-17 Por tôpico Marcelo Souza
Eu gostaria de ver as soluções para o problema:
" Determine o menor inteiro positivo n tal que n =a + b + c, com a,b,c inteiros positivos, de forma que a + b, b + c, a + c sejam quadrados perfeitos"
Obrigado
[]'s M.Get your FREE download of MSN Explorer at http://explorer.msn.com


Re: Geometria plana

2001-08-08 Por tôpico Marcelo Souza


Essa HP  uma obra de arte, alis, eu a recomendo para todos da lista
http://www.cut-the-knot.com/content.html
abrao
M

From: Marcelo Roseira<[EMAIL PROTECTED]> 
Reply-To: [EMAIL PROTECTED] 
To:<[EMAIL PROTECTED]> 
Subject: Geometria plana 
Date: Tue, 7 Aug 2001 09:41:21 -0300 
 
Estou montando uma apostila de Geometria Plana. Algum teria uma boa dica de site (de preferncia em Portugus) com boas listas de exerccios de Geometria. 
J achei alguns interessantes, mas s com teoria. Preciso de exerccios agora. Se algum souber de algum lugar onde posso encontrar talvez uma apostila para download daria uma grande ajuda. 
 
Obrigado. 
Marcelo Roseira. 
 
The Impossible Dream 
 
To dream the impossible dream, 
To fight the unbeatable foe,... 
To right the unrightable wrong,... 
To try, when your arms are too weary, 
To reach the unreachable star! 
This is my Quest to follow that star, 
No matter how hopeless, no matter how far,... 
And I know, if I'll only be true to this glorious quest, 
That my heart will lie peaceful and calm 
 when I'm laid to my rest. 
And the world will be better for this... 
 
Don Quixote in Man of La Mancha 
 
 
Get your FREE download of MSN Explorer at http://explorer.msn.com


problema

2001-07-28 Por tôpico Marcelo Souza

Esse problema aki tem uma solução que eu naum entendi, se alguém pudesse me 
explicar eu seria grato
Dados a chips brancos, b chips pretos, c chips vermelhos em cima de uma 
mesa. Escolhemos dois De cores diferentes e substituimos cada um pelo de 
terceira cor. O processo se repete. Diga a condição que deve haver entre os 
numeros a,b e c para que ao final todos os chips sejam da mesma cor.
A solução começa no caso inicia, (a,b,c) que após o primeiro passo pode 
passar a ser (a+2, b-1, c-1); ou (a-1,b+2,c-1); ou (a-1,b-1,c+2). A solução 
diz que em qquer caso I = a-b mod 3. E diz que b-c=0 mod 3 e
a-c=0 mod 3 são invariantes, naum entendi essa parte, pq eles são congruos a 
zero mod 3???
abraços
M.

_
Get your FREE download of MSN Explorer at http://explorer.msn.com/intl.asp




Re: Questão Colehgio Naval

2001-07-13 Por tôpico Marcelo Souza

Gente, por favor consertem se eu errei em algum passo, estou sem papel.
O numero de diagonais de um polingono é n(n-3)/2, como isso é multiplo de n, 
digamos, K.n, escrevemos:
Kn = n(n-3)/2, cortando o n, obtemos n = 2K + 3, obviamente K é inteiro ( 
pois o numero de diagonais era divisivel por n). Logo, vemos que o cara é 
impar, ou seja o numero de lados é impar.
1. Não pode haver diagonais passando pelo centro , pois o polingono tem um 
número impar de lados.
2. n  pode ser multiplo de 17, basta tomar K = 7, por exemplo;
3. n pode ser cubo perfeito, de modo que um exemplo é K = 12
4. n pode ser primo.
Se eu naum errei as coisas, tipo conclusoes, a resposta é E, se eu tiver 
errado algo, me avisem, é que sem papel, sem traçar, as vezes vvc faz um 
erro na hora de concluir.
[]'s, M.


From: Anselmo Alves de Sousa [EMAIL PROTECTED]
Reply-To: [EMAIL PROTECTED]
To: [EMAIL PROTECTED]
Subject: Questão Colehgio Naval
Date: Fri, 13 Jul 2001 19:43:51 -0300


Olah pessoal! Fiquei muito feliz pelo resultado brasileiro na IMO. Ao mesmo
tempo fiquei triste pois acertei apenas 10 questhoes na OBM. Descobri a OBM
apenas no ano passado e jah estou no terceiro ano. tomei grande gosto pela
matemática e com certeza continuarei a estudar muito e espero aprender
muito.

Dito isto vamos ao que interessa!

Apareceu na prova do Colehgio Naval em 1997:

Considere as afirmativas abaixo sobre um poligono regular de n lados, onde 
o
numero de diagonais eh multiplo de n.

i.  o pligono naum pode ter diagonal que passa pelo seu centro.

ii.  n pode ser multiplo de 17.

iii.  n pode ser um cubo perfeito.

.  n pode ser primo.

Assinale a alternativa correta:

a) Todas as afirmativas são falsas.
b) Apenas a afirmativa ii eh verdadeira.
c) Apenas as afirmativas ii e iii são verdadeiras.
d) Apenas as afirmativas ii, iii e  são verdadeiras.
e) Todas as afirmativas são verdadeiras.

Espero resoluções

Um grande abraço!!  E um alooh para o prfessor Vitor, o cara eh um genio no
calculo

Anselmo
_
Get Your Private, Free E-mail from MSN Hotmail at http://www.hotmail.com.


_
Get Your Private, Free E-mail from MSN Hotmail at http://www.hotmail.com.




Geometria

2001-07-12 Por tôpico Marcelo Souza


Galera, alguém poderia me mandar a solução deste problema?

Mostre que um triangulo acutangulo qualquer pode ser cortado por segmentos 
de retas em 3 partes diferentes de 3 diferentes formas tal que cada parte 
tenha uma linha de simetria.
[]'s, M.
_
Get Your Private, Free E-mail from MSN Hotmail at http://www.hotmail.com.




Re: problema de funçao

2001-07-04 Por tôpico Marcelo Souza

Fala Paulo, certinho?
Poxa, me deram esta questão lah no curso e eu tb achei o enunciado 
confuso...perguntei pro garoto onde ele tinha visto ele disse que passaram 
pra elemeio complicado...eu naum sei dar mais detalhes, escrevi tudo o 
que estava no papel.
abraços
Marcelo


From: Paulo Santa Rita [EMAIL PROTECTED]
Reply-To: [EMAIL PROTECTED]
To: [EMAIL PROTECTED]
Subject: Re: problema de funçao
Date: Wed, 04 Jul 2001 17:00:52

Ola Marcelo,
Tudo Legal ?

Eu devo estar entendo mal a sua questao, pois a faria em 4 passos. Voce 
pode
explicar a questao abaixo com maiores detalhes ? Achei ela legal, pois me
lembrou aspectos teoricos em processos de otimizacao de algoritmos.

Um grande abraco
Paulo Santa Rita
4,1358,04072001


From: Marcelo Souza [EMAIL PROTECTED]
Reply-To: [EMAIL PROTECTED]
To: [EMAIL PROTECTED]
Subject: problema de funçao
Date: Wed, 04 Jul 2001 04:06:52 -

alguém poderia mandar a resolução deste problema pra mim?
Dadas 1000 funçoes lineares f_k (x) = (p_k).x + q_k , k= 1,2,...,1000,
deseja-se obter o valor da composta f(x) =
f_1(f_2(f_3.f_1000(x)...)
no ponto x_0. Sabendo que em cada passo podemos efetuar simultaneamente
qquer numero de operações aritmeticas com pares de numeros obtidos no 
passo
anterior e que no primeiro passo podemos utilizar os numeros p_1,
p_2,,p_1000, q_1,q_2,,1_1000, x_0, podemos afirmar que, o numero 
de
passos necessarios para obter o valor da composta no ponto x não excede a:
a)10
b)12
c)14
d)16
e) 20

valeu
marcelo
_
Get Your Private, Free E-mail from MSN Hotmail at http://www.hotmail.com.


_
Get Your Private, Free E-mail from MSN Hotmail at http://www.hotmail.com.


_
Get Your Private, Free E-mail from MSN Hotmail at http://www.hotmail.com.




Re: LIVROS BONS

2001-07-04 Por tôpico Marcelo Souza


S pra quem quiser saber tb, eu achei uns 5 livros (algebra I) na livraria ciencia e cultura, novinhos, inclusive foi lah que eu comprei o meu.
[]s, MArcelo

From: josimat<[EMAIL PROTECTED]> 
Reply-To: [EMAIL PROTECTED] 
To: OBM<[EMAIL PROTECTED]> 
Subject: LIVROS BONS 
Date: Wed, 4 Jul 2001 15:08:50 -0300 
 
H algum tempo, apareceu aqui nesta lista algum procurando pelos livros do Morgado, E. Wagner e M. Jorge. Vi alguns exemplares (por R$20) novos (lgebra I e Geometria I) na livraria YAN LIVROS, na rua 7 de setembro, 169, Rio, tel. 2262-9347. 
Abraos, Josimar 
Get Your Private, Free E-mail from MSN Hotmail at http://www.hotmail.com.


problema de funçao

2001-07-03 Por tôpico Marcelo Souza

alguém poderia mandar a resolução deste problema pra mim?
Dadas 1000 funçoes lineares f_k (x) = (p_k).x + q_k , k= 1,2,...,1000, 
deseja-se obter o valor da composta f(x) = f_1(f_2(f_3.f_1000(x)...) 
no ponto x_0. Sabendo que em cada passo podemos efetuar simultaneamente 
qquer numero de operações aritmeticas com pares de numeros obtidos no passo 
anterior e que no primeiro passo podemos utilizar os numeros p_1, 
p_2,,p_1000, q_1,q_2,,1_1000, x_0, podemos afirmar que, o numero de 
passos necessarios para obter o valor da composta no ponto x não excede a:
a)10
b)12
c)14
d)16
e) 20

valeu
marcelo
_
Get Your Private, Free E-mail from MSN Hotmail at http://www.hotmail.com.




Re: 24+32=100 ?

2001-06-03 Por tôpico Marcelo Souza

O meu tah dando um troço esquisito, mas eu vou fuçar o meu baú mental pra 
ver se eu consigo lembrar de algo. Você sabe que nós podemos esqcrever os 
números na base decimal da forma:
10^nx_n + 10^(n-1)x_(n-1) + ...+ 10x_1 + x_0
obs:(caso você não saiba, a^b quer dizer a elevado a b)
Onde x_i = 0,1,2,3,4,5,6,7,8,9 (os restos que a base pode deixar qdo 
dividida). (para i natural)
Quando nós mudamos de base, a única coisa que fazemos é trocar o 10 pela 
base pedida. No caso, a base pedida é x:
Como são dois algarismos:
(24)_x = 2x + 4
(32)_x = 3x + 2
(100)_10 = 0 + 0.10 + 1.10^2 = 100
Daí basta resolver a equação
2x + 4 + 3x + 2 = 100
5x = 94
x = 94/5
|Tipo, eu nunca vi a base dando um número racional, por isso achei 
esquisito, mas eu naum sei se pode, eu acho que a base deve ser natural.
Alguém pode me ajudar se eu tiver errado em alguma coisa, faz tanto tempo 
que eu não faço um probleminha destes.
valeu
abraços
Marcelo


From: Eder [EMAIL PROTECTED]
Reply-To: [EMAIL PROTECTED]
To: [EMAIL PROTECTED]
Subject: 24+32=100 ?
Date: Sun, 3 Jun 2001 11:47:00 -0300

24(na base x)+32(na base x)=100(base 10)

x=?

Acho que essa dúvida deve ser meio básica pro pessoal daqui...Mas eu não 
consegui resolver.Que raciocínio usar nessa questão?



_
Get Your Private, Free E-mail from MSN Hotmail at http://www.hotmail.com.




Re: Primos, multiplos e divisores

2001-06-03 Por tôpico Marcelo Souza

Um número p é dito primo qdo seus unicos divisores POSITIVOS são 1 e p.
Isso concerne o conjunto dos inteiros, é clarop, a relação primo ser 
utilizada com frequencia pra naturais é apenas pra encurtar. De fato, os 
divisores de 2 são 2, -2 , 1 e -1. Se naum considerássemos o fato de 
divisores inteiros positivos teriamos que escrever tyudo isso. Um número p é 
dito primo se seus unicos divisores são 1, -1 , p e -p =)
Um número b divide a sendo chamado b de divisor de a e a um múltiplo de b se 
existe um inteiro q tal que a = bq. A noção de multiplo e divisor é uma 
noção circular (corrijam por favor se eu estiver errado);
falow?
Abraços
MArcelo (uin - 57193686)


From: Gustavo Martins [EMAIL PROTECTED]
Reply-To: [EMAIL PROTECTED]
To: [EMAIL PROTECTED]
Subject: Primos, multiplos e divisores
Date: Sun, 3 Jun 2001 16:16:28 -0300

Cada livro fala uma coisa diferente sobre multíplos, divisores e número 
primos. Uns falam que eles só podem pertencer ao conjunto dos naturais; 
outros dizem que é aos inteiros. Afinal, qual a definição certa?

[]s,
Gustavo

_
Get Your Private, Free E-mail from MSN Hotmail at http://www.hotmail.com.




Re: Equipe selecionada para IMO.

2001-05-30 Por tôpico Marcelo Souza


Parabns aos selecionados, Boa sorte para todos e que continuem a representar o BRasil da forma brilhante de sempre.
abraos
Marcelo

From: Olimpiada Brasileira de Matematica<[EMAIL PROTECTED]> 
Reply-To: [EMAIL PROTECTED] 
To: [EMAIL PROTECTED] 
Subject: Equipe selecionada para IMO. 
Date: Tue, 29 May 2001 16:47:07 -0300 
 
Caros amigos da lista: 
 
A equipe que representara ao Brasil na 42a. IMO 
a ser realizada entre os dias 1 a 14 de julho proximo 
em Washington - DC, EE.UU e' a seguinte: 
 
Lider: Prof. Nicolau C. Saldanha (Rio de Janeiro - RJ) 
Vice - Lider: Prof. Antonio Caminha Muniz Neto (Fortaleza - CE) 
 
BRA 1: Alex Correa Abreu (Rio de Janeiro - RJ) 
BRA 2: Carlos Stein Naves de Brito (Goiania - GO) 
BRA 3: Daniel Pinheiro Sobreira (Fortaleza - CE) 
BRA 4: Davi Maximo Alexandrino Nogueira (Fortaleza - CE) 
BRA 5: Humberto Silva Naves (Sao Paulo - SP) 
BRA 6: Thiago Barros Rodrigues Costa (Fortaleza - CE) 
 
 
Abracos, 
 
Nelly. 
 
Get Your Private, Free E-mail from MSN Hotmail at http://www.hotmail.com.


Re: ainda sobre livros...

2001-05-26 Por tôpico Marcelo Souza

que ignorancia nada, fala, é oi marcelo de novo. É o seguinte, alguns livros 
da MIR de geometria são muito cotados, livros de problemas em geral tb o que 
é o forte da MIR), msas estes livros nós so achamos em sebos. Vale a pena 
dar uma  vasculhada. Os titulos eu naum sei de cabeça, mas xa vê!! Eu acho 
que um é Problems in Geometry, sei lah, nem lembro...alguém pode lembrar?
abraços
Marcelo


From: Alexandre Lemos [EMAIL PROTECTED]
Reply-To: [EMAIL PROTECTED]
To: [EMAIL PROTECTED]
Subject: ainda sobre livros...
Date: Sat, 26 May 2001 02:08:57 -0300

oi, gente...

continuando minha questao sobre os livros

desculpem a ignorancia, mas quais sao os livros da MIR?
gostaria exatamente destes livros pra gente grande... ;-)

mais uma coisa: onde posso achar os famosos livros das
competicoes hungaras dos quais o Marcelo Rufino falou?

estou tentando juntar algum material com nivel maior de
dificuldade gostaria de receber ajuda de voces, como
links interessantes ou mesmo provas que pudessem ser
mandadas para mim por email...

abracos a todos... ;-)

_
Get Your Private, Free E-mail from MSN Hotmail at http://www.hotmail.com.




Re: Livros...

2001-05-24 Por tôpico Marcelo Souza

O do MOrgado e do Wagner GEometria I e II. Pra mim é um dos melhores livros 
de geometria pra quem quer começar a estudar de forma disciplinada. O livro 
trata de assuntos interessantes, desde postulados e suas origens até 
inversão geométrica. Depois, com a base do Geometria I e II, vc pode 
procurar uns da MIR de geometria (q é pra gente grande =)
falow!
abraços
Marcelo!!

From: Alexandre Lemos [EMAIL PROTECTED]
Reply-To: [EMAIL PROTECTED]
To: [EMAIL PROTECTED]
Subject: Livros...
Date: Fri, 25 May 2001 00:18:17 -0300

Oi, pessoal

estou procurando um bom livro de geometria, de preferencia com teoria
rapida e bastantes exercicios (exercicios com bom nivel de dificuldade)


abraco a todos...


_
Get Your Private, Free E-mail from MSN Hotmail at http://www.hotmail.com.




Re: Número grande

2001-05-21 Por tôpico Marcelo Souza

Kra, to sem papel na mao, vou ver se consigo tirar daqui mesmo, nao sei se 
vai dar completo, mas se eu errar, alguem por favor me ajude
Seguinte, criterio de divisibilidade por 7, como e muito algarismo da pra 
aplicar este.
Um numero e divisivel por 7 se
(abc) + 10^3(def) + 10^6(ghi) .
10^3 == -1(mod 7)
10^6 == 1 (mod 7)
10^9 == -1 (mod 7)
..
..
conseiderando a,b,c,d algarismos
|(abc) - (def)| + |(ghi) - (jlm)| .
Logo, este numeros e divisivel por 7 se o modulo da soma da diferenca das 
suas casas decimas (a diferenca das casas pares pelas casas impares)
Com0 temos 600 algarismos, da pra separar em 200 grupos de 3, cada um uma 
casa decimal, aplicando aquilo temos
|456 - 123| + |456-123|  + |456 -123|
Isso repete 100 vezes
333 + 333 + ... + 333
100.333 = 33300 == 1 (mod 7)
Ou seja, deixa resto 1 na divisao por 7
Por favor, se eu tiver feito alguma coisa errada me avisem
abraco
marcelo

From: Marcelo Roseira [EMAIL PROTECTED]
Reply-To: [EMAIL PROTECTED]
To: [EMAIL PROTECTED]
Subject: Número grande
Date: Mon, 21 May 2001 11:51:01 -0300

Caros amigos. Como resolver o seguinte problema.

N=123456123456...123456 tem 600 algarismos. Qual o resto da sua divisão por 
7?

Um abraço a todos.

Obs: Sou novo na lista.  Meu nome é Marcelo Roseira.

_
Get Your Private, Free E-mail from MSN Hotmail at http://www.hotmail.com.




Dois problemas - alguém poderia ajudar?

2001-05-20 Por tôpico Marcelo Souza

Oi, alguém poderia me explicar como resolver os problemas abaixo:
- Num retangulo, cujos lados sã de 20 e 25 unidades de comprimento, são 
colocados (sem tocar nas arestas do retangulo) 120 quandrados menores de 1 
unidade de comprimento. Prove que um círculo de diametro 1 pode ser colocado 
no retangulo (novamente sem tocar as arestas do retangulo), tal que não 
tenha nenhum ponto emn comum com os quadrados.

- Cada uma das 9 linhas deivide um quadrado em dois quadriláteros, tal que a 
razão das suas áreas é 2:3. Prove que pelo menos 3 dessas linhas são 
concorrentes.
obrigado
marcelo
_
Get Your Private, Free E-mail from MSN Hotmail at http://www.hotmail.com.




Re: Algebra

2001-05-14 Por tôpico Marcelo Souza

Eu já vi esse prob erm algum lugar\:
Vejamos
5(a^2+b^2) = 8ab
5 [(a+b)^2 - 2ab] = 8ab
5(a+b)^2 - 10ab = 8ab
5(a+b)^2 = 18ab
(a+b) = sqrt(18ab/5)
Para achar a-b é análogo
5[(a-b)^2 + 2ab] = 8ab
5(a-b)^2 + 10ab = 8ab
5(a-b)^2 = -2ab
(a-b) = sqrt(-2ab/5)
Colocando um embaixo do outro (desculpe o termo)

(a+b)/a-b = sqrt [ (18ab/5)/-2ab/5]
cancelando o cinco e o dois vem que
(a+b)/(a-b) = sqrt-9ab
(a+b)/(a-b) = 3i.sqrt(ab)
essa seria uma resposta em C, esquisito, vi um problema muito 
parecidolembrei, no meu livrao =)...mas só que o resultado naum saia 
complexo, era real mesmo...
espero ter ajudado
abraçao
Marcelo


From: Claudio Antonio Teixeira Bastos [EMAIL PROTECTED]
Reply-To: [EMAIL PROTECTED]
To: OBM [EMAIL PROTECTED]
CC: Claudio Bastos [EMAIL PROTECTED]
Subject: Algebra
Date: Mon, 14 May 2001 12:15:37 -0300 (BRT)

Determiner os possíveis valores de (a+b)/(a-b) em C; sabendo que
5a^2+5b^2=8*a*b

_
Get Your Private, Free E-mail from MSN Hotmail at http://www.hotmail.com.




  1   2   >